Tải bản đầy đủ (.doc) (48 trang)

deon thi thu dai hoc

Bạn đang xem bản rút gọn của tài liệu. Xem và tải ngay bản đầy đủ của tài liệu tại đây (283.87 KB, 48 trang )

<span class='text_page_counter'>(1)</span><div class='page_container' data-page=1>

TRƯỜNG THPT LÊ HỒNG PHONG <b> KỲ THI THỬ ĐẠI HỌC - LẦN II</b>
<b> Thị xã Bỉm Sơn </b> <b> NĂM HỌC 2010-2011</b>


<b> </b> <b> </b> <b> Môn thi: Tiếng Anh</b>


<b> Thời gian làm bài: 90 phút</b>
<b> ĐỀ CHÍNH THỨC</b>


<b> (đề thi gồm 5 trang)</b> <b> Mã đề: 121</b>
<b> </b>


<i> Họ và tên thí sinh:... Số báo danh:...</i>


<i><b>I. Mark the letter A, B, C, or D on your answer sheet to indicate the word that differs from the rest in </b></i>
<i><b>the position of the main stress in each of the following sentences.</b></i>


<b>01.A.</b> competitive <b>B.</b> pedestrian <b>C.</b> catalogue <b>D.</b> experienced


<b>02.A.</b> government <b>B.</b> insurance <b>C.</b> envelope <b>D.</b> lavatory


<b>03.A.</b> preferably <b>B.</b> screwdriver <b>C.</b> fortunately <b>D.</b> imprudent


<b>04.A.</b> police <b>B.</b> hotel <b>C.</b> injured <b>D.</b> cassette


<b>05.A.</b> mistake <b>B.</b> nuclear <b>C.</b> machine <b>D.</b> refer


<i><b>II. Read the following passage and mark the letter A, B, C or D on your answer sheet to indicate the </b></i>
<i><b>correct answer to each of the following questions.</b></i>


Lead poisoning in children is a major health concern. Both low and high doses of lead can have serious
effects. Children <b>exposed to</b> high doses of lead often <b>suffer</b> permanent nerve damage, mental retardation,


short attention spans, distractibility, poor academic performance, and behavioural problems. This is not a
new concern. As early as 1904, lead poisoning in children was linked to lead-based paint. Microscopic lead
particles from paint are absorbed into bloodstream when the children ingest flakes of <b>chipped</b> paint, plaster
or paint dust from suckling, or chewing on toys and other objects painted with lead-based paint. Despite
amount of lead used in their paint, this source of leading poisoning is still the most common and most
dangerous. Children living in older, <b>dilapidated</b> houses are particularly <b>at risk</b>.


<b>06.</b> Which of the following is NOT true?


<b>A.</b> Lead poisoning can lead to mental retardation.


<b>B.</b> Only high dose of lead can have serious effects.


<b>C.</b> American paint companies today must comply with strict regulations regarding the amount of lead
used in their paint.


<b>D.</b> Lead poisoning in children was linked to lead-based paint in the 1900s.


<b>07.</b> According to the passage, what is the most common source of lead poisoning in children?


<b>A.</b> lead-based paint <b>B.</b> household dust <b>C.</b> dilapidated houses <b>D.</b> painted toys


<b>08.</b> Which of the following is closest in meaning to the phrase "<b>at risk</b>" ?


<b>A.</b> in safe way <b>B.</b> in danger <b>C.</b> no harm <b>D.</b> in good condition


<b>09.</b> Which of the following is closest in meaning to the word "<b>suffer</b>" ?.


<b>A.</b> feel paint from <b>B.</b> reveal <b>C.</b> experience <b>D.</b> grieve with



<b>10.</b> Which of the following is closest in meaning to "<b>chipped</b>"?


<b>A.</b> unhealthy <b>B.</b> sprayed <b>C.</b> canned <b>D.</b> fragmented


<b>11.</b> The phrase "<b>exposed to</b>" could be best replaced by which of the following?


<b>A.</b> displaying <b>B.</b> conducting <b>C.</b> familiar with <b>D.</b> in contact with


<b>12.</b> What does the author imply in the final sentence of the passage?


<b>A.</b> Old homes were painted with lead-based paint.


<b>B.</b> Lead-based paint chips off more easily than newer paints.


<b>C.</b> Old homes need to be rebuilt in order to be safe for children.


<b>D.</b> Poor people did not comply with the regulations.


<b>13.</b> The word " <b>dilapidated " </b>is closest in meaning to which of the following?


<b>A.</b> broken down <b>B.</b> poorly painted <b>C.</b> fell down <b>D.</b> unpainted


<b>14.</b> Which of the following can be inferred from the passage?


</div>
<span class='text_page_counter'>(2)</span><div class='page_container' data-page=2>

<b>B.</b> Paint companies have always followed regulations regarding the amount of lead used in their paint.


<b>C.</b> Paint companies aren't required to limit the amount lead used in their paint.


<b>D.</b> Paint companies must limit the amount lead used in their paint.



<b>15.</b> What is the main topic of the passage?


<b>A.</b> Lead poisoning in children. <b>B.</b> Problems with household paint.


<b>C.</b> Major health concern for children. <b>D.</b> Lead paint in older houses.


<b>III.Mark the letter A, B, C, or D on your answer sheet to show the underlined part that needs </b>
<b>correction. </b>


<b>16.</b> Vitamin K providing the necessary impetus for the synthesis of at least two proteins involved in
blood clotting.


<b>A.</b> providing <b>B.</b> necessary <b>C.</b> at least <b>D.</b> involved


<b>17.</b> For the past few years, researchers have perfecting their control over the movements of cells and
microbes by using low power laser beams.


<b>A.</b> using <b>B.</b> have perfecting <b>C.</b> their <b>D.</b> few years


<b>18.</b> The nests of most bird species are strategic placed to camouflage them against predators.


<b>A.</b> species <b>B.</b> against predators. <b>C.</b> most <b>D.</b> strategic


<b>19.</b> Perhaps the most unique thing about carbon atoms are their ability to combine with themselves.


<b>A.</b> are <b>B.</b> about <b>C.</b> the most <b>D.</b> to combine


<b>20.</b> Organisms and their cells live by maintaining a constant exchange of elemental, ions, minerals and
gases.



<b>A.</b> gases <b>B.</b> of elemental <b>C.</b> by maintaining <b>D.</b> constant


<i><b>IV. Read the following passage and mark the letter A, B, C or D on your answer sheet to indicate the </b></i>
<i><b>correct answer to each of the following questions.</b></i>


Watching for wildlife in the forests, we rarely see past the surface of things. Standing on the ground
floor, we scan the leafy rafters, entirely overlooking the living world in the soil beneath our feet. The
forest's basement is a secret world. As different from our own world as water is from air, the soil seems
quiet, even dead. But life <b>bustles</b> down below: a cubic inch of top soil may contain billions of creatures.
Predators and prey roam beneath as well as above the forest floor. Furthermore, those upstairs and
downstairs forest denizens live closed <b>linked</b> lives. Soil-dwelling bacteria and fungi break down dead
organic matter into molecules that above ground plants use for food. Those plants as well as animals,
mature and die, leaving more organic matter to fuel the folks downstairs.


Like a well-insulated house, the soil protects its tenants from extreme temperatures, and from rain and
snow. It also provides a <b>bulwark</b> against predators that roam the surface world. But the dense, protecting
soil also limits mobility. Soil creatures must be specially equipped in order to travel easily through their
dark, <b>constricting</b> realm. Earthworms and ants are the champion earth-movers, creating channels that
allow air and water to enter the soil. While ants travel relatively far from their nests, earthworms work
small areas, <b>reprocessing </b>vast amounts of soil into fertile "waste". In a single year, as much as 36 tons of
soil may pass through the alimentary tracts of all the earthworms living in an acre of soil.


<b>21.</b> The word " <b>bulwark</b>" in the passage is closest in meaning to ...


<b>A.</b> radar <b>B.</b> tomb <b>C.</b> barrier <b>D.</b> gateway


<b>22.</b> The word "<b>linked</b>" in the passage is closest in meaning to ...


<b>A.</b> mechanical <b>B.</b> chained <b>C.</b> related <b>D.</b> measured



<b>23.</b> The word "<b>constricting</b>" in the passage is closest in meaning to ...


<b>A.</b> limiting <b>B.</b> damp <b>C.</b> heavy <b>D.</b> deep


<b>24.</b> The word "<b>reprocessing</b>" in the passage is closest in meaning to ...


<b>A.</b> converting <b>B.</b> eliminating <b>C.</b> transporting <b>D.</b> arranging


<b>25.</b> The main topic of the passage is ...


<b>A.</b> the life cycle of ants and worms <b>B.</b> life in the forest soil


<b>C.</b> the habits of the forest animals. <b>D.</b> a description of a forest scene


<b>26.</b> The word "<b>bustles</b>" in the passage is closest in meaning to ...


</div>
<span class='text_page_counter'>(3)</span><div class='page_container' data-page=3>

<b>27.</b> It could be expected that ants ...


<b>A.</b> are more mobile than earthworms <b>B.</b> live only above ground


<b>C.</b> perform similar functions to fungi <b>D.</b> move more earth than earthworms


<b>28.</b> It can be inferred from the passage that the forest soil is ...


<b>A.</b> sparely inhabited <b>B.</b> unknown to man <b>C.</b> densely inhabited <b>D.</b> devoid of life


<b>29.</b> The soil offers creatures that live underground protection EXPECT ...


<b>A.</b> bad weather <b>B.</b> extreme heat and cold <b>C.</b> enemies <b>D.</b> bacteria and fungi



<b>30.</b> According to the passage, what is the main function of bacteria and fungi?


<b>A.</b> to provide food for plant life <b>B.</b> to build walls in the soil


<b>C.</b> to kill mature plants <b>D.</b> to help aerate the soil


<i><b>V. Mark the letter A, B, C or D on your answer sheet to indicate the sentence that is closest in meaning </b></i>
<i><b>to each of following sentences.</b></i>


<b>31.</b> This is the most delicious meat I've ever eaten.


<b>A.</b> Not ever in my life have I had such a wonderful meat.


<b>B.</b> Never in my life have I had such a delicious meat


<b>C.</b> At no time in my life have I never tasted this excellent meat.


<b>D.</b> I have never eaten such more delicious meat.


<b>32.</b> She started work three months ago.


<b>A.</b> It is three months since she started work. <b>B.</b> She is working here for three months now.


<b>C.</b> She had been working for three months. <b>D.</b> It's three months that she worked for.


<b>33.</b> My father finds maps hard to follow.


<b>A.</b> My father always gets lost. <b>B.</b> My father can't read maps at all.


<b>C.</b> Map-reading is not interesting to my father at all. <b>D.</b> My father has trouble following maps.



<b>34.</b> She has lost her appetite recently.


<b>A.</b> Her appetite has been very good. <b>B.</b> She has gone off food recently.


<b>C.</b> She has eaten a lot of food recently. <b>D.</b> She hasn't had any food recently.


<b>35.</b> We came home early to avoid the coming storm.


<b>A.</b> In order to avoiding the coming storm, we came home early.


<b>B.</b> Because it was predicted storming, we decided to come home earlier than unusual.


<b>C.</b> We didn't come home late to not get the coming storm.


<b>D.</b> We came home early in order to avoid the coming storm.


<b>36.</b> My career as a teacher began 14 years ago.


<b>A.</b> For 14 years have I been a teacher. <b>B.</b> I was a teacher for 14 years.


<b>C.</b> I have started teaching for 14 years now. <b>D.</b> I have been a teacher for 14 years now.


<b>37.</b> This affair does not concern you.


<b>A.</b> This affair is no business of yours. <b>B.</b> Your concern is to do this affair.


<b>C.</b> This affair is not interesting. <b>D.</b> Don't do this affair.


<b>38.</b> Not many people attended the meeting.



<b>A.</b> People didn't show up at the meeting. <b>B.</b> There was a poor turn-out for the meeting.


<b>C.</b> Too many people turned out at the meeting. <b>D.</b> No people came to the meeting


<b>39.</b> My father regretted working for that company.


<b>A.</b> My father didn't like working for that company.


<b>B.</b> My father wished he hadn't worked for that company.


<b>C.</b> If only my father was not working for that company.


<b>D.</b> It is my father's regret working for that company.


<b>40.</b> "How beautiful is the dress you have just bought!" Peter said to Mary.


<b>A.</b> Peter said thanks to Mary for her beautiful dress


<b>B.</b> Peter asked Mary how she had just bought her beautiful dress.


<b>C.</b> Peter complimented Mary on her beautiful dress.


</div>
<span class='text_page_counter'>(4)</span><div class='page_container' data-page=4>

<i><b>VI. Mark the letter A, B, C, or D on your answer sheet to indicate the correct answer to each of the </b></i>
<i><b>following sentences</b></i>


<b>41.</b> This film ... several scenes which are very funny.


<b>A.</b> features <b>B.</b> depicts <b>C.</b> illustrates <b>D.</b> pictures



<b>42.</b> The product was withdrawn from sale because there was no longer any ... for it.


<b>A.</b> interest <b>B.</b> order <b>C.</b> call <b>D.</b> claim


<b>43.</b> No one can visit her because she has a very ...disease.


<b>A.</b> corporal <b>B.</b> exasperating <b>C.</b> intoxicating <b>D.</b> contagious


<b>44.</b> She should have been here but she's ... flu.


<b>A.</b> gone down with <b>B.</b> gone through with <b>C.</b> come up against <b>D.</b> come in for


<b>45.</b> By the year 2012, many people currently employed ... their jobs.


<b>A.</b> are losing <b>B.</b> will have lost <b>C.</b> have lost <b>D.</b> will be losing


<b>46.</b> How exactly did you set ...training the horses to work so well together?


<b>A.</b> empty <b>B.</b> to <b>C.</b> loose <b>D.</b> up


<b>47.</b> There's a rumour that the Nation Bank is going to ... the company I work for.


<b>A.</b> take off <b>B.</b> take on <b>C.</b> take over <b>D.</b> overtake


<b>48.</b> " What's the biggest problem in your country?" - " We have ...of energy."


<b>A.</b> the shortage <b>B.</b> a shortage <b>C.</b> some shortage <b>D.</b> shortage


<b>49.</b> "Who broke the window?" - "..."



<b>A.</b> I am <b>B.</b> I was, thanks <b>C.</b> I did. Excuse me. <b>D.</b> I did. Sorry.


<b>50.</b> All the cereal grains ... grow on the prairies and plains of the United States.


<b>A.</b> except the rice <b>B.</b> excepting rice <b>C.</b> but rice <b>D.</b> but for rice


<b>51.</b> "Don't tell anyone my new address" - "..."


<b>A.</b> I can't <b>B.</b> I don't <b>C.</b> I wouldn't <b>D.</b> I won't


<b>52.</b> Automobile ... propane gas emits fewer dangerous pollutants into the atmosphere.


<b>A.</b> that are used <b>B.</b> using <b>C.</b> use <b>D.</b> can use


<b>53.</b> He tried to explain how he felt, but he was unable to ...his true feelings.


<b>A.</b> inhibit <b>B.</b> evolve <b>C.</b> articulate <b>D.</b> inflict


<b>54.</b> Scientists are still uncertain of ... originated millions of years ago.


<b>A.</b> which the universe<b>B.</b> how the universe <b>C.</b> with the universe <b>D.</b> about the universe


<b>55.</b> Only when a great deal of more information has been obtained, ...to plan a manned trip to the
planet.


<b>A.</b> it will probably be <b>B.</b> it will be possible <b>C.</b> will it possible <b>D.</b> it will be likely


<b>56.</b> As the President was absent, I was ask to ... the meeting.


<b>A.</b> officiate <b>B.</b> chair <b>C.</b> regulate <b>D.</b> govern



<b>57.</b> A child in the first grade tends to be ... all of the other children in the class.


<b>A.</b> the same age as <b>B.</b> as old like <b>C.</b> the same old to <b>D.</b> the same age than


<b>58.</b> I'd like to have a bath, but there doesn't seem to be a ... or anything to stop.


<b>A.</b> cork <b>B.</b> lid <b>C.</b> cap <b>D.</b> plug


<b>59.</b> Bigamy is a situation in which a man ... two women at the same time.


<b>A.</b> is married to <b>B.</b> married <b>C.</b> is marry to <b>D.</b> marries to


<b>60.</b> A dog ...on his owner's lap may refuse to eat from a bowl on the floor.


<b>A.</b> fed <b>B.</b> is fed <b>C.</b> was fed <b>D.</b> to feed


<b>61.</b> I had to get up early, ... I'd have missed the train.


<b>A.</b> yet <b>B.</b> if not <b>C.</b> otherwise <b>D.</b> but


<b>62.</b> "Can I use your phone?" - "..."


<b>A.</b> Of course <b>B.</b> You may not <b>C.</b> No, you can't <b>D.</b> You must


<b>63.</b> I'd sooner ...deliver the new furniture tomorrow.


<b>A.</b> mustn't <b>B.</b> wouldn't <b>C.</b> shouldn't <b>D.</b> didn't


<b>64.</b> Technological advances aid in teaching, ... the basic role of teachers stays the same.



</div>
<span class='text_page_counter'>(5)</span><div class='page_container' data-page=5>

<b>65.</b> ...the public's concern about the local environment this new road scheme will have to be
abandoned.


<b>A.</b> As regards <b>B.</b> However much <b>C.</b> In view of <b>D.</b> In the event of


<b>66.</b> - Janet: "Do you feel like going to the cinema tonight?" - Susan: "..."


<b>A.</b> I don't agree, I am afraid. <b>B.</b> I feel very bored.


<b>C.</b> You're welcome <b>D.</b> That would be great


<b>67.</b> ... will remain the foremost world language is considered inevitable by many people.


<b>A.</b> Because English <b>B.</b> That English <b>C.</b> English <b>D.</b> It is English that


<b>68.</b> Their climate is not dry at all; in fact, they have ...of water.


<b>A.</b> a redundancy <b>B.</b> an abundance <b>C.</b> a conglomeration <b>D.</b> a deficiency


<b>69.</b> When another person sneezes, you say "..."


<b>A.</b> Bless you <b>B.</b> Great you <b>C.</b> Pardon? <b>D.</b> See you


<b>70.</b> That popular magazine has many ...


<b>A.</b> prescriptions <b>B.</b> spectators <b>C.</b> subscribers <b>D.</b> denominations
<i><b>VII. Read the following passage and mark the letter A, B, C or D on your answer sheet to indicate the </b></i>
<i><b>correct word for each of the blanks.</b></i>



In this age of ...(71)... telephone networks and electronic mail, it seems that fewer and even fewer
people are taking time to sit down and write letters ...(72)... friends and relatives. For hundreds of years,
letters were the only way to keep ...(73)... people who were any distance away and letter-writing was seen
as an important skill for all learned people...(74).... Gradually, ...(75)..., the importance of writing letters is
decreasing to a point that majority of us have to ...(76)... a special effort to turn out something worthwhile
when we apply for a job or make an appointment. In business circles the tendency is for routine
communications to become shorter. ...(77)...clients may appreciate a detailed letter, an employee who
sends out long letter is often regarded as ...(78).... Many people prefer the telephone in all circumstances
and its speed is essential in many situations but ...(79)... have you put the telephone down, dissatisfied
with what you have managed to say? I don't think I'll throw my ...(80)... away yet.


<b>71.A.</b> advanced <b>B.</b> progressive <b>C.</b> all are correct <b>D.</b> highly-developed


<b>72.A.</b> with <b>B.</b> for <b>C.</b> to <b>D.</b> from


<b>73.A.</b> in step with <b>B.</b> on good terms with <b>C.</b> in favour of <b>D.</b> in contact with


<b>74.A.</b> mastered <b>B.</b> mastering <b>C.</b> to master <b>D.</b> to be mastered


<b>75.A.</b> in short <b>B.</b> for example <b>C.</b> therefore <b>D.</b> however


<b>76.A.</b> cause <b>B.</b> make <b>C.</b> do <b>D.</b> create


<b>77.A.</b> Even though <b>B.</b> As though <b>C.</b> Despite <b>D.</b> However


<b>78.A.</b> unusual <b>B.</b> inefficient <b>C.</b> unimportant <b>D.</b> impossible


<b>79.A.</b> how much <b>B.</b> how about <b>C.</b> how often <b>D.</b> how long


</div>
<span class='text_page_counter'>(6)</span><div class='page_container' data-page=6>

<i>Đáp án đề: 121</i>



<b> 01. </b>- - } - <b> 21. </b>- - } - <b> 41. </b>{ - - - <b> 61. </b>- - } -


<b> 02. </b>- | - - <b> 22. </b>- - } - <b> 42. </b>- - } - <b> 62. </b>{ - - -


<b> 03. </b>- - - ~ <b> 23. </b>{ - - - <b> 43. </b>- - - ~ <b> 63. </b>- - } -


<b> 04. </b>- | - - <b> 24. </b>{ - - - <b> 44. </b>{ - - - <b> 64. </b>- - - ~


<b> 05. </b>- | - - <b> 25. </b>- | - - <b> 45. </b>- | - - <b> 65. </b>- - } -


<b> 06. </b>- | - - <b> 26. </b>{ - - - <b> 46. </b>- - - ~ <b> 66. </b>- - - ~


<b> 07. </b>{ - - - <b> 27. </b>{ - - - <b> 47. </b>- - } - <b> 67. </b>- | - -


<b> 08. </b>- | - - <b> 28. </b>- - } - <b> 48. </b>- | - - <b> 68. </b>- | - -


<b> 09. </b>- - } - <b> 29. </b>- - - ~ <b> 49. </b>- - - ~ <b> 69. </b>{ - - -


<b> 10. </b>- - - ~ <b> 30. </b>{ - - - <b> 50. </b>- - } - <b> 70. </b>- - } -


<b> 11. </b>- - - ~ <b> 31. </b>- - - ~ <b> 51. </b>- - - ~ <b> 71. </b>- - } -


<b> 12. </b>- - } - <b> 32. </b>{ - - - <b> 52. </b>- | - - <b> 72. </b>- - } -


<b> 13. </b>{ - - - <b> 33. </b>- - - ~ <b> 53. </b>- - } - <b> 73. </b>- - - ~


<b> 14. </b>- - - ~ <b> 34. </b>- | - - <b> 54. </b>- | - - <b> 74. </b>- - } -


<b> 15. </b>{ - - - <b> 35. </b>- - - ~ <b> 55. </b>- - } - <b> 75. </b>- - - ~



<b> 16. </b>{ - - - <b> 36. </b>- - - ~ <b> 56. </b>- | - - <b> 76. </b>- | - -


<b> 17. </b>- | - - <b> 37. </b>{ - - - <b> 57. </b>{ - - - <b> 77. </b>{ - - -


<b> 18. </b>- - - ~ <b> 38. </b>- | - - <b> 58. </b>- - - ~ <b> 78. </b>- | - -


<b> 19. </b>{ - - - <b> 39. </b>- | - - <b> 59. </b>{ - - - <b> 79. </b>- - } -


</div>
<span class='text_page_counter'>(7)</span><div class='page_container' data-page=7>

TRƯỜNG THPT LÊ HỒNG PHONG <b> KỲ THI THỬ ĐẠI HỌC - LẦN II</b>
<b> Thị xã Bỉm Sơn </b> <b> NĂM HỌC 2010-2011</b>


<b> </b> <b> </b> <b> Môn thi: Tiếng Anh</b>


<b> Thời gian làm bài: 90 phút</b>
<b> ĐỀ CHÍNH THỨC</b>


<b> (đề thi gồm 5 trang)</b> <b> Mã đề: 122</b>
<b> </b>


<i> Họ và tên thí sinh:... Số báo danh:...</i>


<i><b>I. Read the following passage and mark the letter A, B, C or D on your answer sheet to indicate the </b></i>
<i><b>correct word for each of the blanks.</b></i>


In this age of ...(1)... telephone networks and electronic mail, it seems that fewer and even fewer
people are taking time to sit down and write letters ...(2)... friends and relatives. For hundreds of years,
letters were the only way to keep ...(3)... people who were any distance away and letter-writing was seen as
an important skill for all learned people...(4).... Gradually, ...(5)..., the importance of writing letters is
decreasing to a point that majority of us have to ...(6)... a special effort to turn out something worthwhile


when we apply for a job or make an appointment. In business circles the tendency is for routine
communications to become shorter. ...(7)...clients may appreciate a detailed letter, an employee who sends
out long letter is often regarded as ...(8).... Many people prefer the telephone in all circumstances and its
speed is essential in many situations but ...(9)... have you put the telephone down, dissatisfied with what
you have managed to say? I don't think I'll throw my ...(10)... away yet.


<b>01.A.</b> progressive <b>B.</b> all are correct <b>C.</b> highly-developed <b>D.</b> advanced


<b>02.A.</b> with <b>B.</b> for <b>C.</b> from <b>D.</b> to


<b>03.A.</b> on good terms with <b>B.</b> in favour of <b>C.</b> in step with <b>D.</b> in contact with


<b>04.A.</b> mastering <b>B.</b> to be mastered <b>C.</b> mastered <b>D.</b> to master


<b>05.A.</b> in short <b>B.</b> for example <b>C.</b> however <b>D.</b> therefore


<b>06.A.</b> cause <b>B.</b> do <b>C.</b> create <b>D.</b> make


<b>07.A.</b> Even though <b>B.</b> As though <b>C.</b> Despite <b>D.</b> However


<b>08.A.</b> impossible <b>B.</b> unimportant <b>C.</b> inefficient <b>D.</b> unusual


<b>09.A.</b> how long <b>B.</b> how often <b>C.</b> how much <b>D.</b> how about


<b>10.A.</b> effort <b>B.</b> telephone <b>C.</b> pen <b>D.</b> letter


<i><b>II. Read the following passage and mark the letter A, B, C or D on your answer sheet to indicate the </b></i>
<i><b>correct answer to each of the following questions.</b></i>


Lead poisoning in children is a major health concern. Both low and high doses of lead can have serious


effects. Children <b>exposed to</b> high doses of lead often <b>suffer</b> permanent nerve damage, mental retardation,
short attention spans, distractibility, poor academic performance, and behavioural problems. This is not a
new concern. As early as 1904, lead poisoning in children was linked to lead-based paint. Microscopic lead
particles from paint are absorbed into bloodstream when the children ingest flakes of <b>chipped</b> paint, plaster
or paint dust from suckling, or chewing on toys and other objects painted with lead-based paint. Despite
amount of lead used in their paint, this source of leading poisoning is still the most common and most
dangerous. Children living in older, <b>dilapidated</b> houses are particularly <b>at risk</b>.


<b>11.</b> What is the main topic of the passage?


<b>A.</b> Lead paint in older houses. <b>B.</b> Major health concern for children.


<b>C.</b> Problems with household paint. <b>D.</b> Lead poisoning in children.


<b>12.</b> What does the author imply in the final sentence of the passage?


<b>A.</b> Old homes need to be rebuilt in order to be safe for children.


<b>B.</b> Lead-based paint chips off more easily than newer paints.


<b>C.</b> Poor people did not comply with the regulations.


<b>D.</b> Old homes were painted with lead-based paint.


<b>13.</b> Which of the following can be inferred from the passage?


</div>
<span class='text_page_counter'>(8)</span><div class='page_container' data-page=8>

<b>B.</b> Paint companies can no longer use lead in their paint.


<b>C.</b> Paint companies must limit the amount lead used in their paint.



<b>D.</b> Paint companies aren't required to limit the amount lead used in their paint.


<b>14.</b> According to the passage, what is the most common source of lead poisoning in children?


<b>A.</b> dilapidated houses <b>B.</b> lead-based paint <b>C.</b> household dust <b>D.</b> painted toys


<b>15.</b> The word " <b>dilapidated " </b>is closest in meaning to which of the following?


<b>A.</b> poorly painted <b>B.</b> unpainted <b>C.</b> broken down <b>D.</b> fell down


<b>16.</b> Which of the following is closest in meaning to "<b>chipped</b>"?


<b>A.</b> canned <b>B.</b> sprayed <b>C.</b> fragmented <b>D.</b> unhealthy


<b>17.</b> The phrase "<b>exposed to</b>" could be best replaced by which of the following?


<b>A.</b> displaying <b>B.</b> in contact with <b>C.</b> familiar with <b>D.</b> conducting


<b>18.</b> Which of the following is closest in meaning to the phrase "<b>at risk</b>" ?


<b>A.</b> in safe way <b>B.</b> no harm <b>C.</b> in good condition <b>D.</b> in danger


<b>19.</b> Which of the following is closest in meaning to the word "<b>suffer</b>" ?.


<b>A.</b> experience <b>B.</b> grieve with <b>C.</b> feel paint from <b>D.</b> reveal


<b>20.</b> Which of the following is NOT true?


<b>A.</b> Lead poisoning can lead to mental retardation.



<b>B.</b> American paint companies today must comply with strict regulations regarding the amount of lead
used in their paint.


<b>C.</b> Lead poisoning in children was linked to lead-based paint in the 1900s.


<b>D.</b> Only high dose of lead can have serious effects.


<i><b>III. Mark the letter A, B, C, or D on your answer sheet to indicate the correct answer to each of the </b></i>
<i><b>following sentences</b></i>


<b>21.</b> Technological advances aid in teaching, ... the basic role of teachers stays the same.


<b>A.</b> but <b>B.</b> with <b>C.</b> because of <b>D.</b> despite


<b>22.</b> ...the public's concern about the local environment this new road scheme will have to be
abandoned.


<b>A.</b> However much <b>B.</b> As regards <b>C.</b> In the event of <b>D.</b> In view of


<b>23.</b> He tried to explain how he felt, but he was unable to ...his true feelings.


<b>A.</b> inflict <b>B.</b> articulate <b>C.</b> evolve <b>D.</b> inhibit


<b>24.</b> Only when a great deal of more information has been obtained, ...to plan a manned trip to the
planet.


<b>A.</b> it will be likely <b>B.</b> will it possible <b>C.</b> it will be possible <b>D.</b> it will probably be


<b>25.</b> Their climate is not dry at all; in fact, they have ...of water.



<b>A.</b> a deficiency <b>B.</b> a redundancy <b>C.</b> a conglomeration <b>D.</b> an abundance


<b>26.</b> "Who broke the window?" - "..."


<b>A.</b> I was, thanks <b>B.</b> I am <b>C.</b> I did. Excuse me. <b>D.</b> I did. Sorry.


<b>27.</b> "Don't tell anyone my new address" - "..."


<b>A.</b> I don't <b>B.</b> I can't <b>C.</b> I wouldn't <b>D.</b> I won't


<b>28.</b> When another person sneezes, you say "..."


<b>A.</b> Bless you <b>B.</b> Great you <b>C.</b> See you <b>D.</b> Pardon?


<b>29.</b> I'd like to have a bath, but there doesn't seem to be a ... or anything to stop.


<b>A.</b> cap <b>B.</b> cork <b>C.</b> lid <b>D.</b> plug


<b>30.</b> How exactly did you set ...training the horses to work so well together?


<b>A.</b> empty <b>B.</b> to <b>C.</b> loose <b>D.</b> up


<b>31.</b> This film ... several scenes which are very funny.


<b>A.</b> illustrates <b>B.</b> features <b>C.</b> depicts <b>D.</b> pictures


<b>32.</b> By the year 2012, many people currently employed ... their jobs.


<b>A.</b> are losing <b>B.</b> have lost <b>C.</b> will be losing <b>D.</b> will have lost



<b>33.</b> Bigamy is a situation in which a man ... two women at the same time.


</div>
<span class='text_page_counter'>(9)</span><div class='page_container' data-page=9>

<b>34.</b> A dog ...on his owner's lap may refuse to eat from a bowl on the floor.


<b>A.</b> is fed <b>B.</b> fed <b>C.</b> was fed <b>D.</b> to feed


<b>35.</b> I had to get up early, ... I'd have missed the train.


<b>A.</b> if not <b>B.</b> otherwise <b>C.</b> yet <b>D.</b> but


<b>36.</b> As the President was absent, I was ask to ... the meeting.


<b>A.</b> officiate <b>B.</b> regulate <b>C.</b> chair <b>D.</b> govern


<b>37.</b> There's a rumour that the Nation Bank is going to ... the company I work for.


<b>A.</b> overtake <b>B.</b> take off <b>C.</b> take on <b>D.</b> take over


<b>38.</b> Automobile ... propane gas emits fewer dangerous pollutants into the atmosphere.


<b>A.</b> can use <b>B.</b> that are used <b>C.</b> using <b>D.</b> use


<b>39.</b> ... will remain the foremost world language is considered inevitable by many people.


<b>A.</b> Because English <b>B.</b> That English <b>C.</b> It is English that <b>D.</b> English


<b>40.</b> No one can visit her because she has a very ...disease.


<b>A.</b> corporal <b>B.</b> contagious <b>C.</b> exasperating <b>D.</b> intoxicating



<b>41.</b> Scientists are still uncertain of ... originated millions of years ago.


<b>A.</b> with the universe <b>B.</b> which the universe <b>C.</b> how the universe <b>D.</b> about the universe


<b>42.</b> - Janet: "Do you feel like going to the cinema tonight?" - Susan: "..."


<b>A.</b> I feel very bored. <b>B.</b> You're welcome


<b>C.</b> That would be great <b>D.</b> I don't agree, I am afraid.


<b>43.</b> All the cereal grains ... grow on the prairies and plains of the United States.


<b>A.</b> but rice <b>B.</b> excepting rice <b>C.</b> but for rice <b>D.</b> except the rice


<b>44.</b> I'd sooner ...deliver the new furniture tomorrow.


<b>A.</b> wouldn't <b>B.</b> mustn't <b>C.</b> shouldn't <b>D.</b> didn't


<b>45.</b> A child in the first grade tends to be ... all of the other children in the class.


<b>A.</b> the same old to <b>B.</b> the same age than <b>C.</b> as old like <b>D.</b> the same age as


<b>46.</b> The product was withdrawn from sale because there was no longer any ... for it.


<b>A.</b> interest <b>B.</b> call <b>C.</b> order <b>D.</b> claim


<b>47.</b> "Can I use your phone?" - "..."


<b>A.</b> No, you can't <b>B.</b> You must <b>C.</b> Of course <b>D.</b> You may not



<b>48.</b> " What's the biggest problem in your country?" - " We have ...of energy."


<b>A.</b> a shortage <b>B.</b> some shortage <b>C.</b> shortage <b>D.</b> the shortage


<b>49.</b> She should have been here but she's ... flu.


<b>A.</b> come in for <b>B.</b> gone through with <b>C.</b> gone down with <b>D.</b> come up against


<b>50.</b> That popular magazine has many ...


<b>A.</b> prescriptions <b>B.</b> spectators <b>C.</b> denominations <b>D.</b> subscribers


<i><b>IV. Mark the letter A, B, C, or D on your answer sheet to indicate the word that differs from the rest in </b></i>
<i><b>the position of the main stress in each of the following sentences.</b></i>


<b>51.A.</b> mistake <b>B.</b> machine <b>C.</b> refer <b>D.</b> nuclear


<b>52.A.</b> screwdriver <b>B.</b> preferably <b>C.</b> imprudent <b>D.</b> fortunately


<b>53.A.</b> injured <b>B.</b> hotel <b>C.</b> cassette <b>D.</b> police


<b>54.A.</b> envelope <b>B.</b> insurance <b>C.</b> government <b>D.</b> lavatory


<b>55.A.</b> experienced <b>B.</b> catalogue <b>C.</b> competitive <b>D.</b> pedestrian


<i><b>V. Mark the letter A, B, C, or D on your answer sheet to show the underlined part that needs correction.</b></i>


<b>56.</b> Vitamin K providing the necessary impetus for the synthesis of at least two proteins involved in
blood clotting.



<b>A.</b> providing <b>B.</b> involved <b>C.</b> necessary <b>D.</b> at least


<b>57.</b> For the past few years, researchers have perfecting their control over the movements of cells and
microbes by using low power laser beams.


<b>A.</b> using <b>B.</b> their <b>C.</b> have perfecting <b>D.</b> few years


</div>
<span class='text_page_counter'>(10)</span><div class='page_container' data-page=10>

<b>A.</b> most <b>B.</b> species <b>C.</b> strategic <b>D.</b> against predators.


<b>59.</b> Perhaps the most unique thing about carbon atoms are their ability to combine with themselves.


<b>A.</b> the most <b>B.</b> are <b>C.</b> to combine <b>D.</b> about


<b>60.</b> Organisms and their cells live by maintaining a constant exchange of elemental, ions, minerals and
gases.


<b>A.</b> constant <b>B.</b> by maintaining <b>C.</b> gases <b>D.</b> of elemental


<i><b>VI. Mark the letter A, B, C or D on your answer sheet to indicate the sentence that is closest in meaning </b></i>
<i><b>to each of following sentences.</b></i>


<b>61.</b> This is the most delicious meat I've ever eaten.


<b>A.</b> At no time in my life have I never tasted this excellent meat.


<b>B.</b> Not ever in my life have I had such a wonderful meat.


<b>C.</b> Never in my life have I had such a delicious meat


<b>D.</b> I have never eaten such more delicious meat.



<b>62.</b> Not many people attended the meeting.


<b>A.</b> No people came to the meeting <b>B.</b> People didn't show up at the meeting.


<b>C.</b> Too many people turned out at the meeting. <b>D.</b> There was a poor turn-out for the meeting.


<b>63.</b> She started work three months ago.


<b>A.</b> She had been working for three months. <b>B.</b> It is three months since she started work.


<b>C.</b> She is working here for three months now. <b>D.</b> It's three months that she worked for.


<b>64.</b> My father finds maps hard to follow.


<b>A.</b> Map-reading is not interesting to my father at all. <b>B.</b> My father always gets lost.


<b>C.</b> My father can't read maps at all. <b>D.</b> My father has trouble following maps.


<b>65.</b> We came home early to avoid the coming storm.


<b>A.</b> Because it was predicted storming, we decided to come home earlier than unusual.


<b>B.</b> In order to avoiding the coming storm, we came home early.


<b>C.</b> We came home early in order to avoid the coming storm.


<b>D.</b> We didn't come home late to not get the coming storm.


<b>66.</b> This affair does not concern you.



<b>A.</b> This affair is not interesting. <b>B.</b> This affair is no business of yours.


<b>C.</b> Your concern is to do this affair. <b>D.</b> Don't do this affair.


<b>67.</b> "How beautiful is the dress you have just bought!" Peter said to Mary.


<b>A.</b> Peter complimented Mary on her beautiful dress.


<b>B.</b> Peter promised to buy Mary a beautiful dress.


<b>C.</b> Peter asked Mary how she had just bought her beautiful dress.


<b>D.</b> Peter said thanks to Mary for her beautiful dress


<b>68.</b> My career as a teacher began 14 years ago.


<b>A.</b> For 14 years have I been a teacher. <b>B.</b> I have started teaching for 14 years now.


<b>C.</b> I have been a teacher for 14 years now. <b>D.</b> I was a teacher for 14 years.


<b>69.</b> My father regretted working for that company.


<b>A.</b> It is my father's regret working for that company.


<b>B.</b> My father didn't like working for that company.


<b>C.</b> If only my father was not working for that company.


<b>D.</b> My father wished he hadn't worked for that company.



<b>70.</b> She has lost her appetite recently.


<b>A.</b> She has gone off food recently. <b>B.</b> She hasn't had any food recently.


<b>C.</b> Her appetite has been very good. <b>D.</b> She has eaten a lot of food recently.


<i><b>VII. Read the following passage and mark the letter A, B, C or D on your answer sheet to indicate the </b></i>
<i><b>correct answer to each of the following questions.</b></i>


</div>
<span class='text_page_counter'>(11)</span><div class='page_container' data-page=11>

forest's basement is a secret world. As different from our own world as water is from air, the soil seems
quiet, even dead. But life <b>bustles</b> down below: a cubic inch of top soil may contain billions of creatures.
Predators and prey roam beneath as well as above the forest floor. Furthermore, those upstairs and
downstairs forest denizens live closed <b>linked</b> lives. Soil-dwelling bacteria and fungi break down dead
organic matter into molecules that above ground plants use for food. Those plants as well as animals,
mature and die, leaving more organic matter to fuel the folks downstairs.


Like a well-insulated house, the soil protects its tenants from extreme temperatures, and from rain and
snow. It also provides a <b>bulwark</b> against predators that roam the surface world. But the dense, protecting
soil also limits mobility. Soil creatures must be specially equipped in order to travel easily through their
dark, <b>constricting</b> realm. Earthworms and ants are the champion earth-movers, creating channels that
allow air and water to enter the soil. While ants travel relatively far from their nests, earthworms work
small areas, <b>reprocessing </b>vast amounts of soil into fertile "waste". In a single year, as much as 36 tons of
soil may pass through the alimentary tracts of all the earthworms living in an acre of soil.


<b>71.</b> The word "<b>linked</b>" in the passage is closest in meaning to ...


<b>A.</b> mechanical <b>B.</b> related <b>C.</b> measured <b>D.</b> chained


<b>72.</b> It could be expected that ants ...



<b>A.</b> live only above ground <b>B.</b> move more earth than earthworms


<b>C.</b> are more mobile than earthworms <b>D.</b> perform similar functions to fungi


<b>73.</b> The word "<b>bustles</b>" in the passage is closest in meaning to ...


<b>A.</b> continues <b>B.</b> waits <b>C.</b> is very active <b>D.</b> threatens


<b>74.</b> The main topic of the passage is ...


<b>A.</b> life in the forest soil <b>B.</b> a description of a forest scene


<b>C.</b> the habits of the forest animals. <b>D.</b> the life cycle of ants and worms


<b>75.</b> The soil offers creatures that live underground protection EXPECT ...


<b>A.</b> extreme heat and cold <b>B.</b> bacteria and fungi <b>C.</b> bad weather <b>D.</b> enemies


<b>76.</b> The word " <b>bulwark</b>" in the passage is closest in meaning to ...


<b>A.</b> gateway <b>B.</b> barrier <b>C.</b> tomb <b>D.</b> radar


<b>77.</b> It can be inferred from the passage that the forest soil is ...


<b>A.</b> devoid of life <b>B.</b> densely inhabited <b>C.</b> sparely inhabited <b>D.</b> unknown to man


<b>78.</b> According to the passage, what is the main function of bacteria and fungi?


<b>A.</b> to build walls in the soil <b>B.</b> to help aerate the soil



<b>C.</b> to provide food for plant life <b>D.</b> to kill mature plants


<b>79.</b> The word "<b>constricting</b>" in the passage is closest in meaning to ...


<b>A.</b> heavy <b>B.</b> deep <b>C.</b> limiting <b>D.</b> damp


<b>80.</b> The word "<b>reprocessing</b>" in the passage is closest in meaning to ...


</div>
<span class='text_page_counter'>(12)</span><div class='page_container' data-page=12>

<i>Đáp án đề: 122</i>


<b> 01. </b>- | - - <b> 21. </b>{ - - - <b> 41. </b>- - } - <b> 61. </b>- - - ~


<b> 02. </b>- - - ~ <b> 22. </b>- - - ~ <b> 42. </b>- - } - <b> 62. </b>- - - ~


<b> 03. </b>- - - ~ <b> 23. </b>- | - - <b> 43. </b>{ - - - <b> 63. </b>- | - -


<b> 04. </b>- - - ~ <b> 24. </b>- | - - <b> 44. </b>- - } - <b> 64. </b>- - - ~


<b> 05. </b>- - } - <b> 25. </b>- - - ~ <b> 45. </b>- - - ~ <b> 65. </b>- - } -


<b> 06. </b>- - - ~ <b> 26. </b>- - - ~ <b> 46. </b>- | - - <b> 66. </b>- | - -


<b> 07. </b>{ - - - <b> 27. </b>- - - ~ <b> 47. </b>- - } - <b> 67. </b>{ - - -


<b> 08. </b>- - } - <b> 28. </b>{ - - - <b> 48. </b>{ - - - <b> 68. </b>- - } -


<b> 09. </b>- | - - <b> 29. </b>- - - ~ <b> 49. </b>- - } - <b> 69. </b>- - - ~


<b> 10. </b>- - } - <b> 30. </b>- - - ~ <b> 50. </b>- - - ~ <b> 70. </b>{ - - -



<b> 11. </b>- - - ~ <b> 31. </b>- | - - <b> 51. </b>- - - ~ <b> 71. </b>- | - -


<b> 12. </b>{ - - - <b> 32. </b>- - - ~ <b> 52. </b>- - } - <b> 72. </b>- - } -


<b> 13. </b>- - } - <b> 33. </b>- - } - <b> 53. </b>- | - - <b> 73. </b>- - } -


<b> 14. </b>- | - - <b> 34. </b>- | - - <b> 54. </b>- | - - <b> 74. </b>{ - - -


<b> 15. </b>- - } - <b> 35. </b>- | - - <b> 55. </b>- | - - <b> 75. </b>- | - -


<b> 16. </b>- - } - <b> 36. </b>- - } - <b> 56. </b>{ - - - <b> 76. </b>- | - -


<b> 17. </b>- | - - <b> 37. </b>- - - ~ <b> 57. </b>- - } - <b> 77. </b>- | - -


<b> 18. </b>- - - ~ <b> 38. </b>- - } - <b> 58. </b>- - } - <b> 78. </b>- - } -


<b> 19. </b>{ - - - <b> 39. </b>- | - - <b> 59. </b>- | - - <b> 79. </b>- - } -


</div>
<span class='text_page_counter'>(13)</span><div class='page_container' data-page=13>

TRƯỜNG THPT LÊ HỒNG PHONG <b> KỲ THI THỬ ĐẠI HỌC - LẦN II</b>
<b> Thị xã Bỉm Sơn </b> <b> NĂM HỌC 2010-2011</b>


<b> </b> <b> </b> <b> Môn thi: Tiếng Anh</b>


<b> Thời gian làm bài: 90 phút</b>
<b> ĐỀ CHÍNH THỨC</b>


<b> (đề thi gồm 5 trang)</b> <b> Mã đề: 123</b>
<b> </b>



<i> Họ và tên thí sinh:... Số báo danh:...</i>


<i><b>I. Read the following passage and mark the letter A, B, C or D on your answer sheet to indicate the </b></i>
<i><b>correct answer to each of the following questions.</b></i>


Lead poisoning in children is a major health concern. Both low and high doses of lead can have serious
effects. Children <b>exposed to</b> high doses of lead often <b>suffer</b> permanent nerve damage, mental retardation,
short attention spans, distractibility, poor academic performance, and behavioural problems. This is not a
new concern. As early as 1904, lead poisoning in children was linked to lead-based paint. Microscopic lead
particles from paint are absorbed into bloodstream when the children ingest flakes of <b>chipped</b> paint, plaster
or paint dust from suckling, or chewing on toys and other objects painted with lead-based paint. Despite
amount of lead used in their paint, this source of leading poisoning is still the most common and most
dangerous. Children living in older, <b>dilapidated</b> houses are particularly <b>at risk</b>.


<b>01.</b> What is the main topic of the passage?


<b>A.</b> Lead paint in older houses. <b>B.</b> Major health concern for children.


<b>C.</b> Problems with household paint. <b>D.</b> Lead poisoning in children.


<b>02.</b> Which of the following is closest in meaning to the word "<b>suffer</b>" ?.


<b>A.</b> reveal <b>B.</b> feel paint from <b>C.</b> grieve with <b>D.</b> experience


<b>03.</b> Which of the following is closest in meaning to "<b>chipped</b>"?


<b>A.</b> sprayed <b>B.</b> fragmented <b>C.</b> unhealthy <b>D.</b> canned


<b>04.</b> The phrase "<b>exposed to</b>" could be best replaced by which of the following?



<b>A.</b> familiar with <b>B.</b> displaying <b>C.</b> in contact with <b>D.</b> conducting


<b>05.</b> According to the passage, what is the most common source of lead poisoning in children?


<b>A.</b> painted toys <b>B.</b> lead-based paint <b>C.</b> dilapidated houses <b>D.</b> household dust


<b>06.</b> Which of the following is NOT true?


<b>A.</b> Only high dose of lead can have serious effects.


<b>B.</b> American paint companies today must comply with strict regulations regarding the amount of lead
used in their paint.


<b>C.</b> Lead poisoning in children was linked to lead-based paint in the 1900s.


<b>D.</b> Lead poisoning can lead to mental retardation.


<b>07.</b> Which of the following is closest in meaning to the phrase "<b>at risk</b>" ?


<b>A.</b> in danger <b>B.</b> in good condition <b>C.</b> no harm <b>D.</b> in safe way


<b>08.</b> The word " <b>dilapidated " </b>is closest in meaning to which of the following?


<b>A.</b> unpainted <b>B.</b> broken down <b>C.</b> poorly painted <b>D.</b> fell down


<b>09.</b> Which of the following can be inferred from the passage?


<b>A.</b> Paint companies must limit the amount lead used in their paint.


<b>B.</b> Paint companies have always followed regulations regarding the amount of lead used in their paint.



<b>C.</b> Paint companies can no longer use lead in their paint.


<b>D.</b> Paint companies aren't required to limit the amount lead used in their paint.


<b>10.</b> What does the author imply in the final sentence of the passage?


<b>A.</b> Poor people did not comply with the regulations.


<b>B.</b> Old homes need to be rebuilt in order to be safe for children.


<b>C.</b> Old homes were painted with lead-based paint.


</div>
<span class='text_page_counter'>(14)</span><div class='page_container' data-page=14>

<i><b>II. Mark the letter A, B, C, or D on your answer sheet to indicate the correct answer to each of the </b></i>
<i><b>following sentences</b></i>


<b>11.</b> I'd sooner ...deliver the new furniture tomorrow.


<b>A.</b> mustn't <b>B.</b> wouldn't <b>C.</b> didn't <b>D.</b> shouldn't


<b>12.</b> Technological advances aid in teaching, ... the basic role of teachers stays the same.


<b>A.</b> despite <b>B.</b> because of <b>C.</b> with <b>D.</b> but


<b>13.</b> As the President was absent, I was ask to ... the meeting.


<b>A.</b> officiate <b>B.</b> chair <b>C.</b> govern <b>D.</b> regulate


<b>14.</b> The product was withdrawn from sale because there was no longer any ... for it.



<b>A.</b> order <b>B.</b> claim <b>C.</b> interest <b>D.</b> call


<b>15.</b> This film ... several scenes which are very funny.


<b>A.</b> pictures <b>B.</b> features <b>C.</b> depicts <b>D.</b> illustrates


<b>16.</b> No one can visit her because she has a very ...disease.


<b>A.</b> contagious <b>B.</b> corporal <b>C.</b> exasperating <b>D.</b> intoxicating


<b>17.</b> How exactly did you set ...training the horses to work so well together?


<b>A.</b> up <b>B.</b> empty <b>C.</b> loose <b>D.</b> to


<b>18.</b> ...the public's concern about the local environment this new road scheme will have to be
abandoned.


<b>A.</b> In view of <b>B.</b> As regards <b>C.</b> In the event of <b>D.</b> However much


<b>19.</b> Bigamy is a situation in which a man ... two women at the same time.


<b>A.</b> marries to <b>B.</b> married <b>C.</b> is marry to <b>D.</b> is married to


<b>20.</b> I'd like to have a bath, but there doesn't seem to be a ... or anything to stop.


<b>A.</b> plug <b>B.</b> cap <b>C.</b> cork <b>D.</b> lid


<b>21.</b> Scientists are still uncertain of ... originated millions of years ago.


<b>A.</b> about the universe <b>B.</b> with the universe <b>C.</b> which the universe <b>D.</b> how the universe



<b>22.</b> There's a rumour that the Nation Bank is going to ... the company I work for.


<b>A.</b> overtake <b>B.</b> take over <b>C.</b> take on <b>D.</b> take off


<b>23.</b> I had to get up early, ... I'd have missed the train.


<b>A.</b> but <b>B.</b> otherwise <b>C.</b> yet <b>D.</b> if not


<b>24.</b> ... will remain the foremost world language is considered inevitable by many people.


<b>A.</b> Because English <b>B.</b> It is English that <b>C.</b> English <b>D.</b> That English


<b>25.</b> "Who broke the window?" - "..."


<b>A.</b> I was, thanks <b>B.</b> I did. Excuse me. <b>C.</b> I did. Sorry. <b>D.</b> I am


<b>26.</b> All the cereal grains ... grow on the prairies and plains of the United States.


<b>A.</b> but for rice <b>B.</b> but rice <b>C.</b> excepting rice <b>D.</b> except the rice


<b>27.</b> By the year 2012, many people currently employed ... their jobs.


<b>A.</b> will be losing <b>B.</b> have lost <b>C.</b> are losing <b>D.</b> will have lost


<b>28.</b> - Janet: "Do you feel like going to the cinema tonight?" - Susan: "..."


<b>A.</b> You're welcome <b>B.</b> I don't agree, I am afraid.


<b>C.</b> That would be great <b>D.</b> I feel very bored.



<b>29.</b> Only when a great deal of more information has been obtained, ...to plan a manned trip to the
planet.


<b>A.</b> it will probably be <b>B.</b> it will be likely <b>C.</b> it will be possible <b>D.</b> will it possible


<b>30.</b> "Don't tell anyone my new address" - "..."


<b>A.</b> I can't <b>B.</b> I don't <b>C.</b> I won't <b>D.</b> I wouldn't


<b>31.</b> Automobile ... propane gas emits fewer dangerous pollutants into the atmosphere.


<b>A.</b> using <b>B.</b> that are used <b>C.</b> can use <b>D.</b> use


<b>32.</b> She should have been here but she's ... flu.


<b>A.</b> come up against <b>B.</b> gone through with <b>C.</b> gone down with <b>D.</b> come in for


<b>33.</b> "Can I use your phone?" - "..."


<b>A.</b> You must <b>B.</b> No, you can't <b>C.</b> You may not <b>D.</b> Of course


</div>
<span class='text_page_counter'>(15)</span><div class='page_container' data-page=15>

<b>A.</b> Great you <b>B.</b> Pardon? <b>C.</b> See you <b>D.</b> Bless you


<b>35.</b> A dog ...on his owner's lap may refuse to eat from a bowl on the floor.


<b>A.</b> is fed <b>B.</b> to feed <b>C.</b> was fed <b>D.</b> fed


<b>36.</b> " What's the biggest problem in your country?" - " We have ...of energy."



<b>A.</b> shortage <b>B.</b> some shortage <b>C.</b> a shortage <b>D.</b> the shortage


<b>37.</b> He tried to explain how he felt, but he was unable to ...his true feelings.


<b>A.</b> evolve <b>B.</b> articulate <b>C.</b> inflict <b>D.</b> inhibit


<b>38.</b> Their climate is not dry at all; in fact, they have ...of water.


<b>A.</b> an abundance <b>B.</b> a deficiency <b>C.</b> a redundancy <b>D.</b> a conglomeration


<b>39.</b> A child in the first grade tends to be ... all of the other children in the class.


<b>A.</b> the same age than <b>B.</b> the same old to <b>C.</b> the same age as <b>D.</b> as old like


<b>40.</b> That popular magazine has many ...


<b>A.</b> denominations <b>B.</b> subscribers <b>C.</b> prescriptions <b>D.</b> spectators


<i><b>III. Mark the letter A, B, C, or D on your answer sheet to indicate the word that differs from the rest in </b></i>
<i><b>the position of the main stress in each of the following sentences.</b></i>


<b>41.A.</b> hotel <b>B.</b> police <b>C.</b> injured <b>D.</b> cassette


<b>42.A.</b> imprudent <b>B.</b> screwdriver <b>C.</b> fortunately <b>D.</b> preferably


<b>43.A.</b> lavatory <b>B.</b> insurance <b>C.</b> government <b>D.</b> envelope


<b>44.A.</b> catalogue <b>B.</b> pedestrian <b>C.</b> experienced <b>D.</b> competitive


<b>45.A.</b> machine <b>B.</b> mistake <b>C.</b> nuclear <b>D.</b> refer



<i><b>IV. Read the following passage and mark the letter A, B, C or D on your answer sheet to indicate the </b></i>
<i><b>correct word for each of the blanks.</b></i>


In this age of ...(46)... telephone networks and electronic mail, it seems that fewer and even fewer
people are taking time to sit down and write letters ...(47)... friends and relatives. For hundreds of years,
letters were the only way to keep ...(48)... people who were any distance away and letter-writing was seen
as an important skill for all learned people...(49).... Gradually, ...(50)..., the importance of writing letters is
decreasing to a point that majority of us have to ...(51)... a special effort to turn out something worthwhile
when we apply for a job or make an appointment. In business circles the tendency is for routine
communications to become shorter. ...(52)...clients may appreciate a detailed letter, an employee who
sends out long letter is often regarded as ...(53).... Many people prefer the telephone in all circumstances
and its speed is essential in many situations but ...(54)... have you put the telephone down, dissatisfied
with what you have managed to say? I don't think I'll throw my ...(55)... away yet.


<b>46.A.</b> all are correct <b>B.</b> highly-developed <b>C.</b> advanced <b>D.</b> progressive


<b>47.A.</b> for <b>B.</b> with <b>C.</b> to <b>D.</b> from


<b>48.A.</b> in contact with <b>B.</b> in favour of <b>C.</b> in step with <b>D.</b> on good terms with


<b>49.A.</b> mastered <b>B.</b> to master <b>C.</b> to be mastered <b>D.</b> mastering


<b>50.A.</b> therefore <b>B.</b> in short <b>C.</b> however <b>D.</b> for example


<b>51.A.</b> cause <b>B.</b> do <b>C.</b> create <b>D.</b> make


<b>52.A.</b> Despite <b>B.</b> As though <b>C.</b> However <b>D.</b> Even though


<b>53.A.</b> impossible <b>B.</b> inefficient <b>C.</b> unimportant <b>D.</b> unusual



<b>54.A.</b> how long <b>B.</b> how often <b>C.</b> how much <b>D.</b> how about


<b>55.A.</b> letter <b>B.</b> telephone <b>C.</b> effort <b>D.</b> pen


<i><b>V. Read the following passage and mark the letter A, B, C or D on your answer sheet to indicate the </b></i>
<i><b>correct answer to each of the following questions.</b></i>


</div>
<span class='text_page_counter'>(16)</span><div class='page_container' data-page=16>

downstairs forest denizens live closed <b>linked</b> lives. Soil-dwelling bacteria and fungi break down dead
organic matter into molecules that above ground plants use for food. Those plants as well as animals,
mature and die, leaving more organic matter to fuel the folks downstairs.


Like a well-insulated house, the soil protects its tenants from extreme temperatures, and from rain and
snow. It also provides a <b>bulwark</b> against predators that roam the surface world. But the dense, protecting
soil also limits mobility. Soil creatures must be specially equipped in order to travel easily through their
dark, <b>constricting</b> realm. Earthworms and ants are the champion earth-movers, creating channels that
allow air and water to enter the soil. While ants travel relatively far from their nests, earthworms work
small areas, <b>reprocessing </b>vast amounts of soil into fertile "waste". In a single year, as much as 36 tons of
soil may pass through the alimentary tracts of all the earthworms living in an acre of soil.


<b>56.</b> It could be expected that ants ...


<b>A.</b> move more earth than earthworms <b>B.</b> live only above ground


<b>C.</b> are more mobile than earthworms <b>D.</b> perform similar functions to fungi


<b>57.</b> According to the passage, what is the main function of bacteria and fungi?


<b>A.</b> to kill mature plants <b>B.</b> to build walls in the soil



<b>C.</b> to provide food for plant life <b>D.</b> to help aerate the soil


<b>58.</b> The main topic of the passage is ...


<b>A.</b> the habits of the forest animals. <b>B.</b> a description of a forest scene


<b>C.</b> the life cycle of ants and worms <b>D.</b> life in the forest soil


<b>59.</b> The soil offers creatures that live underground protection EXPECT ...


<b>A.</b> bacteria and fungi <b>B.</b> extreme heat and cold <b>C.</b> bad weather <b>D.</b> enemies


<b>60.</b> The word "<b>linked</b>" in the passage is closest in meaning to ...


<b>A.</b> mechanical <b>B.</b> chained <b>C.</b> related <b>D.</b> measured


<b>61.</b> It can be inferred from the passage that the forest soil is ...


<b>A.</b> devoid of life <b>B.</b> densely inhabited <b>C.</b> unknown to man <b>D.</b> sparely inhabited


<b>62.</b> The word "<b>constricting</b>" in the passage is closest in meaning to ...


<b>A.</b> damp <b>B.</b> heavy <b>C.</b> limiting <b>D.</b> deep


<b>63.</b> The word "<b>bustles</b>" in the passage is closest in meaning to ...


<b>A.</b> threatens <b>B.</b> is very active <b>C.</b> waits <b>D.</b> continues


<b>64.</b> The word " <b>bulwark</b>" in the passage is closest in meaning to ...



<b>A.</b> gateway <b>B.</b> barrier <b>C.</b> radar <b>D.</b> tomb


<b>65.</b> The word "<b>reprocessing</b>" in the passage is closest in meaning to ...


<b>A.</b> eliminating <b>B.</b> converting <b>C.</b> transporting <b>D.</b> arranging
<i><b>VI. Mark the letter A, B, C, or D on your answer sheet to show the underlined part that needs </b></i>
<i><b>correction. </b></i>


<b>66.</b> Vitamin K providing the necessary impetus for the synthesis of at least two proteins involved in
blood clotting.


<b>A.</b> at least <b>B.</b> necessary <b>C.</b> involved <b>D.</b> providing


<b>67.</b> The nests of most bird species are strategic placed to camouflage them against predators.


<b>A.</b> species <b>B.</b> against predators. <b>C.</b> strategic <b>D.</b> most


<b>68.</b> For the past few years, researchers have perfecting their control over the movements of cells and
microbes by using low power laser beams.


<b>A.</b> few years <b>B.</b> their <b>C.</b> using <b>D.</b> have perfecting


<b>69.</b> Perhaps the most unique thing about carbon atoms are their ability to combine with themselves.


<b>A.</b> the most <b>B.</b> about <b>C.</b> are <b>D.</b> to combine


<b>70.</b> Organisms and their cells live by maintaining a constant exchange of elemental, ions, minerals and
gases.


<b>A.</b> gases <b>B.</b> constant <b>C.</b> of elemental <b>D.</b> by maintaining



</div>
<span class='text_page_counter'>(17)</span><div class='page_container' data-page=17>

<b>71.</b> She has lost her appetite recently.


<b>A.</b> She has eaten a lot of food recently. <b>B.</b> Her appetite has been very good.


<b>C.</b> She hasn't had any food recently. <b>D.</b> She has gone off food recently.


<b>72.</b> We came home early to avoid the coming storm.


<b>A.</b> We didn't come home late to not get the coming storm.


<b>B.</b> Because it was predicted storming, we decided to come home earlier than unusual.


<b>C.</b> We came home early in order to avoid the coming storm.


<b>D.</b> In order to avoiding the coming storm, we came home early.


<b>73.</b> "How beautiful is the dress you have just bought!" Peter said to Mary.


<b>A.</b> Peter asked Mary how she had just bought her beautiful dress.


<b>B.</b> Peter said thanks to Mary for her beautiful dress


<b>C.</b> Peter complimented Mary on her beautiful dress.


<b>D.</b> Peter promised to buy Mary a beautiful dress.


<b>74.</b> This affair does not concern you.


<b>A.</b> Your concern is to do this affair. <b>B.</b> This affair is no business of yours.



<b>C.</b> This affair is not interesting. <b>D.</b> Don't do this affair.


<b>75.</b> She started work three months ago.


<b>A.</b> She is working here for three months now. <b>B.</b> It's three months that she worked for.


<b>C.</b> It is three months since she started work. <b>D.</b> She had been working for three months.


<b>76.</b> My career as a teacher began 14 years ago.


<b>A.</b> For 14 years have I been a teacher. <b>B.</b> I was a teacher for 14 years.


<b>C.</b> I have been a teacher for 14 years now. <b>D.</b> I have started teaching for 14 years now.


<b>77.</b> My father regretted working for that company.


<b>A.</b> My father wished he hadn't worked for that company.


<b>B.</b> If only my father was not working for that company.


<b>C.</b> My father didn't like working for that company.


<b>D.</b> It is my father's regret working for that company.


<b>78.</b> This is the most delicious meat I've ever eaten.


<b>A.</b> I have never eaten such more delicious meat.


<b>B.</b> Not ever in my life have I had such a wonderful meat.



<b>C.</b> At no time in my life have I never tasted this excellent meat.


<b>D.</b> Never in my life have I had such a delicious meat


<b>79.</b> My father finds maps hard to follow.


<b>A.</b> My father always gets lost. <b>B.</b> My father can't read maps at all.


<b>C.</b> Map-reading is not interesting to my father at all. <b>D.</b> My father has trouble following maps.


<b>80.</b> Not many people attended the meeting.


<b>A.</b> Too many people turned out at the meeting. <b>B.</b> There was a poor turn-out for the meeting.


</div>
<span class='text_page_counter'>(18)</span><div class='page_container' data-page=18>

<i>Đáp án đề: 123</i>


<b> 01. </b>- - - ~ <b> 21. </b>- - - ~ <b> 41. </b>{ - - - <b> 61. </b>- | - -


<b> 02. </b>- - - ~ <b> 22. </b>- | - - <b> 42. </b>{ - - - <b> 62. </b>- - } -


<b> 03. </b>- | - - <b> 23. </b>- | - - <b> 43. </b>- | - - <b> 63. </b>- | - -


<b> 04. </b>- - } - <b> 24. </b>- - - ~ <b> 44. </b>{ - - - <b> 64. </b>- | - -


<b> 05. </b>- | - - <b> 25. </b>- - } - <b> 45. </b>- - } - <b> 65. </b>- | - -


<b> 06. </b>{ - - - <b> 26. </b>- | - - <b> 46. </b>{ - - - <b> 66. </b>- - - ~


<b> 07. </b>{ - - - <b> 27. </b>- - - ~ <b> 47. </b>- - } - <b> 67. </b>- - } -



<b> 08. </b>- | - - <b> 28. </b>- - } - <b> 48. </b>{ - - - <b> 68. </b>- - - ~


<b> 09. </b>{ - - - <b> 29. </b>- - - ~ <b> 49. </b>- | - - <b> 69. </b>- - } -


<b> 10. </b>- | - - <b> 30. </b>- - } - <b> 50. </b>- - } - <b> 70. </b>- - } -


<b> 11. </b>- - - ~ <b> 31. </b>{ - - - <b> 51. </b>- - - ~ <b> 71. </b>- - - ~


<b> 12. </b>- - - ~ <b> 32. </b>- - } - <b> 52. </b>- - - ~ <b> 72. </b>- - } -


<b> 13. </b>- | - - <b> 33. </b>- - - ~ <b> 53. </b>- | - - <b> 73. </b>- - } -


<b> 14. </b>- - - ~ <b> 34. </b>- - - ~ <b> 54. </b>- | - - <b> 74. </b>- | - -


<b> 15. </b>- | - - <b> 35. </b>- - - ~ <b> 55. </b>- - - ~ <b> 75. </b>- - } -


<b> 16. </b>{ - - - <b> 36. </b>- - } - <b> 56. </b>- - } - <b> 76. </b>- - } -


<b> 17. </b>{ - - - <b> 37. </b>- | - - <b> 57. </b>- - } - <b> 77. </b>{ - - -


<b> 18. </b>{ - - - <b> 38. </b>{ - - - <b> 58. </b>- - - ~ <b> 78. </b>{ - - -


<b> 19. </b>- - - ~ <b> 39. </b>- - } - <b> 59. </b>{ - - - <b> 79. </b>- - - ~


</div>
<span class='text_page_counter'>(19)</span><div class='page_container' data-page=19>

TRƯỜNG THPT LÊ HỒNG PHONG <b> KỲ THI THỬ ĐẠI HỌC - LẦN II</b>
<b> Thị xã Bỉm Sơn </b> <b> NĂM HỌC 2010-2011</b>


<b> </b> <b> </b> <b> Môn thi: Tiếng Anh</b>



<b> Thời gian làm bài: 90 phút</b>
<b> ĐỀ CHÍNH THỨC</b>


<b> (đề thi gồm 5 trang)</b> <b> Mã đề: 124</b>
<b> </b>


<i> Họ và tên thí sinh:... Số báo danh:...</i>


<i><b>I. Mark the letter A, B, C or D on your answer sheet to indicate the sentence that is closest in meaning </b></i>
<i><b>to each of following sentences.</b></i>


<b>01.</b> "How beautiful is the dress you have just bought!" Peter said to Mary.


<b>A.</b> Peter asked Mary how she had just bought her beautiful dress.


<b>B.</b> Peter promised to buy Mary a beautiful dress.


<b>C.</b> Peter complimented Mary on her beautiful dress.


<b>D.</b> Peter said thanks to Mary for her beautiful dress


<b>02.</b> My father finds maps hard to follow.


<b>A.</b> Map-reading is not interesting to my father at all. <b>B.</b> My father can't read maps at all.


<b>C.</b> My father has trouble following maps. <b>D.</b> My father always gets lost.


<b>03.</b> My career as a teacher began 14 years ago.


<b>A.</b> I have started teaching for 14 years now. <b>B.</b> For 14 years have I been a teacher.



<b>C.</b> I have been a teacher for 14 years now. <b>D.</b> I was a teacher for 14 years.


<b>04.</b> This is the most delicious meat I've ever eaten.


<b>A.</b> I have never eaten such more delicious meat.


<b>B.</b> Not ever in my life have I had such a wonderful meat.


<b>C.</b> At no time in my life have I never tasted this excellent meat.


<b>D.</b> Never in my life have I had such a delicious meat


<b>05.</b> This affair does not concern you.


<b>A.</b> This affair is no business of yours. <b>B.</b> Don't do this affair.


<b>C.</b> This affair is not interesting. <b>D.</b> Your concern is to do this affair.


<b>06.</b> We came home early to avoid the coming storm.


<b>A.</b> We didn't come home late to not get the coming storm.


<b>B.</b> In order to avoiding the coming storm, we came home early.


<b>C.</b> We came home early in order to avoid the coming storm.


<b>D.</b> Because it was predicted storming, we decided to come home earlier than unusual.


<b>07.</b> My father regretted working for that company.



<b>A.</b> My father wished he hadn't worked for that company.


<b>B.</b> My father didn't like working for that company.


<b>C.</b> It is my father's regret working for that company.


<b>D.</b> If only my father was not working for that company.


<b>08.</b> She started work three months ago.


<b>A.</b> She had been working for three months. <b>B.</b> She is working here for three months now.


<b>C.</b> It's three months that she worked for. <b>D.</b> It is three months since she started work.


<b>09.</b> Not many people attended the meeting.


<b>A.</b> People didn't show up at the meeting. <b>B.</b> No people came to the meeting


<b>C.</b> There was a poor turn-out for the meeting. <b>D.</b> Too many people turned out at the meeting.


<b>10.</b> She has lost her appetite recently.


<b>A.</b> She hasn't had any food recently. <b>B.</b> She has eaten a lot of food recently.


<b>C.</b> She has gone off food recently. <b>D.</b> Her appetite has been very good.


</div>
<span class='text_page_counter'>(20)</span><div class='page_container' data-page=20>

<b>11.</b> Bigamy is a situation in which a man ... two women at the same time.


<b>A.</b> is married to <b>B.</b> married <b>C.</b> is marry to <b>D.</b> marries to



<b>12.</b> "Don't tell anyone my new address" - "..."


<b>A.</b> I don't <b>B.</b> I can't <b>C.</b> I won't <b>D.</b> I wouldn't


<b>13.</b> - Janet: "Do you feel like going to the cinema tonight?" - Susan: "..."


<b>A.</b> That would be great <b>B.</b> I don't agree, I am afraid.


<b>C.</b> You're welcome <b>D.</b> I feel very bored.


<b>14.</b> As the President was absent, I was ask to ... the meeting.


<b>A.</b> regulate <b>B.</b> chair <b>C.</b> govern <b>D.</b> officiate


<b>15.</b> He tried to explain how he felt, but he was unable to ...his true feelings.


<b>A.</b> inhibit <b>B.</b> evolve <b>C.</b> inflict <b>D.</b> articulate


<b>16.</b> A dog ...on his owner's lap may refuse to eat from a bowl on the floor.


<b>A.</b> fed <b>B.</b> was fed <b>C.</b> is fed <b>D.</b> to feed


<b>17.</b> There's a rumour that the Nation Bank is going to ... the company I work for.


<b>A.</b> overtake <b>B.</b> take off <b>C.</b> take over <b>D.</b> take on


<b>18.</b> Scientists are still uncertain of ... originated millions of years ago.


<b>A.</b> with the universe <b>B.</b> about the universe <b>C.</b> how the universe <b>D.</b> which the universe



<b>19.</b> By the year 2012, many people currently employed ... their jobs.


<b>A.</b> will have lost <b>B.</b> are losing <b>C.</b> have lost <b>D.</b> will be losing


<b>20.</b> Their climate is not dry at all; in fact, they have ...of water.


<b>A.</b> an abundance <b>B.</b> a redundancy <b>C.</b> a conglomeration <b>D.</b> a deficiency


<b>21.</b> I had to get up early, ... I'd have missed the train.


<b>A.</b> if not <b>B.</b> but <b>C.</b> yet <b>D.</b> otherwise


<b>22.</b> I'd sooner ...deliver the new furniture tomorrow.


<b>A.</b> wouldn't <b>B.</b> shouldn't <b>C.</b> didn't <b>D.</b> mustn't


<b>23.</b> All the cereal grains ... grow on the prairies and plains of the United States.


<b>A.</b> but for rice <b>B.</b> excepting rice <b>C.</b> except the rice <b>D.</b> but rice


<b>24.</b> ...the public's concern about the local environment this new road scheme will have to be
abandoned.


<b>A.</b> In the event of <b>B.</b> However much <b>C.</b> In view of <b>D.</b> As regards


<b>25.</b> ... will remain the foremost world language is considered inevitable by many people.


<b>A.</b> It is English that <b>B.</b> Because English <b>C.</b> English <b>D.</b> That English



<b>26.</b> The product was withdrawn from sale because there was no longer any ... for it.


<b>A.</b> claim <b>B.</b> call <b>C.</b> interest <b>D.</b> order


<b>27.</b> When another person sneezes, you say "..."


<b>A.</b> See you <b>B.</b> Bless you <b>C.</b> Great you <b>D.</b> Pardon?


<b>28.</b> She should have been here but she's ... flu.


<b>A.</b> come up against <b>B.</b> gone down with <b>C.</b> gone through with <b>D.</b> come in for


<b>29.</b> A child in the first grade tends to be ... all of the other children in the class.


<b>A.</b> the same age than <b>B.</b> the same old to <b>C.</b> the same age as <b>D.</b> as old like


<b>30.</b> Only when a great deal of more information has been obtained, ...to plan a manned trip to the
planet.


<b>A.</b> will it possible <b>B.</b> it will be likely <b>C.</b> it will probably be <b>D.</b> it will be possible


<b>31.</b> Automobile ... propane gas emits fewer dangerous pollutants into the atmosphere.


<b>A.</b> using <b>B.</b> can use <b>C.</b> that are used <b>D.</b> use


<b>32.</b> "Can I use your phone?" - "..."


<b>A.</b> No, you can't <b>B.</b> You must <b>C.</b> Of course <b>D.</b> You may not


<b>33.</b> Technological advances aid in teaching, ... the basic role of teachers stays the same.



<b>A.</b> because of <b>B.</b> with <b>C.</b> despite <b>D.</b> but


<b>34.</b> That popular magazine has many ...


<b>A.</b> spectators <b>B.</b> subscribers <b>C.</b> denominations <b>D.</b> prescriptions


</div>
<span class='text_page_counter'>(21)</span><div class='page_container' data-page=21>

<b>A.</b> to <b>B.</b> loose <b>C.</b> up <b>D.</b> empty


<b>36.</b> " What's the biggest problem in your country?" - " We have ...of energy."


<b>A.</b> a shortage <b>B.</b> some shortage <b>C.</b> shortage <b>D.</b> the shortage


<b>37.</b> No one can visit her because she has a very ...disease.


<b>A.</b> corporal <b>B.</b> intoxicating <b>C.</b> contagious <b>D.</b> exasperating


<b>38.</b> I'd like to have a bath, but there doesn't seem to be a ... or anything to stop.


<b>A.</b> cap <b>B.</b> cork <b>C.</b> lid <b>D.</b> plug


<b>39.</b> This film ... several scenes which are very funny.


<b>A.</b> features <b>B.</b> depicts <b>C.</b> illustrates <b>D.</b> pictures


<b>40.</b> "Who broke the window?" - "..."


<b>A.</b> I did. Excuse me. <b>B.</b> I am <b>C.</b> I was, thanks <b>D.</b> I did. Sorry.
<i><b>III. Mark the letter A, B, C, or D on your answer sheet to show the underlined part that needs </b></i>
<i><b>correction. </b></i>



<b>41.</b> For the past few years, researchers have perfecting their control over the movements of cells and
microbes by using low power laser beams.


<b>A.</b> few years <b>B.</b> have perfecting <b>C.</b> their <b>D.</b> using


<b>42.</b> Vitamin K providing the necessary impetus for the synthesis of at least two proteins involved in
blood clotting.


<b>A.</b> at least <b>B.</b> necessary <b>C.</b> involved <b>D.</b> providing


<b>43.</b> Organisms and their cells live by maintaining a constant exchange of elemental, ions, minerals and
gases.


<b>A.</b> by maintaining <b>B.</b> constant <b>C.</b> gases <b>D.</b> of elemental


<b>44.</b> The nests of most bird species are strategic placed to camouflage them against predators.


<b>A.</b> strategic <b>B.</b> against predators. <b>C.</b> most <b>D.</b> species


<b>45.</b> Perhaps the most unique thing about carbon atoms are their ability to combine with themselves.


<b>A.</b> about <b>B.</b> are <b>C.</b> the most <b>D.</b> to combine


<i><b>IV. Read the following passage and mark the letter A, B, C or D on your answer sheet to indicate the </b></i>
<i><b>correct word for each of the blanks.</b></i>


In this age of ...(46)... telephone networks and electronic mail, it seems that fewer and even fewer
people are taking time to sit down and write letters ...(47)... friends and relatives. For hundreds of years,
letters were the only way to keep ...(48)... people who were any distance away and letter-writing was seen


as an important skill for all learned people...(49).... Gradually, ...(50)..., the importance of writing letters is
decreasing to a point that majority of us have to ...(51)... a special effort to turn out something worthwhile
when we apply for a job or make an appointment. In business circles the tendency is for routine
communications to become shorter. ...(52)...clients may appreciate a detailed letter, an employee who
sends out long letter is often regarded as ...(53).... Many people prefer the telephone in all circumstances
and its speed is essential in many situations but ...(54)... have you put the telephone down, dissatisfied
with what you have managed to say? I don't think I'll throw my ...(55)... away yet.


<b>46.A.</b> all are correct <b>B.</b> advanced <b>C.</b> highly-developed <b>D.</b> progressive


<b>47.A.</b> for <b>B.</b> from <b>C.</b> with <b>D.</b> to


<b>48.A.</b> in step with <b>B.</b> in favour of <b>C.</b> on good terms with <b>D.</b> in contact with


<b>49.A.</b> mastering <b>B.</b> to be mastered <b>C.</b> to master <b>D.</b> mastered


<b>50.A.</b> therefore <b>B.</b> in short <b>C.</b> for example <b>D.</b> however


<b>51.A.</b> create <b>B.</b> do <b>C.</b> make <b>D.</b> cause


<b>52.A.</b> Despite <b>B.</b> However <b>C.</b> As though <b>D.</b> Even though


<b>53.A.</b> unimportant <b>B.</b> unusual <b>C.</b> inefficient <b>D.</b> impossible


<b>54.A.</b> how about <b>B.</b> how often <b>C.</b> how much <b>D.</b> how long


</div>
<span class='text_page_counter'>(22)</span><div class='page_container' data-page=22>

<i><b>V. Mark the letter A, B, C, or D on your answer sheet to indicate the word that differs from the rest in </b></i>
<i><b>the position of the main stress in each of the following sentences.</b></i>


<b>56.A.</b> catalogue <b>B.</b> competitive <b>C.</b> experienced <b>D.</b> pedestrian



<b>57.A.</b> screwdriver <b>B.</b> preferably <b>C.</b> imprudent <b>D.</b> fortunately


<b>58.A.</b> mistake <b>B.</b> refer <b>C.</b> machine <b>D.</b> nuclear


<b>59.A.</b> lavatory <b>B.</b> insurance <b>C.</b> envelope <b>D.</b> government


<b>60.A.</b> injured <b>B.</b> hotel <b>C.</b> police <b>D.</b> cassette


<i><b>VI. Read the following passage and mark the letter A, B, C or D on your answer sheet to indicate the </b></i>
<i><b>correct answer to each of the following questions.</b></i>


Lead poisoning in children is a major health concern. Both low and high doses of lead can have serious
effects. Children <b>exposed to</b> high doses of lead often <b>suffer</b> permanent nerve damage, mental retardation,
short attention spans, distractibility, poor academic performance, and behavioural problems. This is not a
new concern. As early as 1904, lead poisoning in children was linked to lead-based paint. Microscopic lead
particles from paint are absorbed into bloodstream when the children ingest flakes of <b>chipped</b> paint, plaster
or paint dust from suckling, or chewing on toys and other objects painted with lead-based paint. Despite
amount of lead used in their paint, this source of leading poisoning is still the most common and most
dangerous. Children living in older, <b>dilapidated</b> houses are particularly <b>at risk</b>.


<b>61.</b> What is the main topic of the passage?


<b>A.</b> Lead paint in older houses. <b>B.</b> Lead poisoning in children.


<b>C.</b> Major health concern for children. <b>D.</b> Problems with household paint.


<b>62.</b> Which of the following is closest in meaning to "<b>chipped</b>"?


<b>A.</b> unhealthy <b>B.</b> canned <b>C.</b> sprayed <b>D.</b> fragmented



<b>63.</b> What does the author imply in the final sentence of the passage?


<b>A.</b> Poor people did not comply with the regulations.


<b>B.</b> Lead-based paint chips off more easily than newer paints.


<b>C.</b> Old homes need to be rebuilt in order to be safe for children.


<b>D.</b> Old homes were painted with lead-based paint.


<b>64.</b> The word " <b>dilapidated " </b>is closest in meaning to which of the following?


<b>A.</b> unpainted <b>B.</b> fell down <b>C.</b> poorly painted <b>D.</b> broken down


<b>65.</b> Which of the following is NOT true?


<b>A.</b> Lead poisoning can lead to mental retardation.


<b>B.</b> Lead poisoning in children was linked to lead-based paint in the 1900s.


<b>C.</b> Only high dose of lead can have serious effects.


<b>D.</b> American paint companies today must comply with strict regulations regarding the amount of lead
used in their paint.


<b>66.</b> Which of the following is closest in meaning to the word "<b>suffer</b>" ?.


<b>A.</b> reveal <b>B.</b> grieve with <b>C.</b> feel paint from <b>D.</b> experience



<b>67.</b> According to the passage, what is the most common source of lead poisoning in children?


<b>A.</b> lead-based paint <b>B.</b> dilapidated houses <b>C.</b> household dust <b>D.</b> painted toys


<b>68.</b> Which of the following can be inferred from the passage?


<b>A.</b> Paint companies must limit the amount lead used in their paint.


<b>B.</b> Paint companies have always followed regulations regarding the amount of lead used in their paint.


<b>C.</b> Paint companies can no longer use lead in their paint.


<b>D.</b> Paint companies aren't required to limit the amount lead used in their paint.


<b>69.</b> Which of the following is closest in meaning to the phrase "<b>at risk</b>" ?


<b>A.</b> no harm <b>B.</b> in danger <b>C.</b> in safe way <b>D.</b> in good condition


<b>70.</b> The phrase "<b>exposed to</b>" could be best replaced by which of the following?


<b>A.</b> in contact with <b>B.</b> displaying <b>C.</b> familiar with <b>D.</b> conducting


</div>
<span class='text_page_counter'>(23)</span><div class='page_container' data-page=23>

Watching for wildlife in the forests, we rarely see past the surface of things. Standing on the ground
floor, we scan the leafy rafters, entirely overlooking the living world in the soil beneath our feet. The
forest's basement is a secret world. As different from our own world as water is from air, the soil seems
quiet, even dead. But life <b>bustles</b> down below: a cubic inch of top soil may contain billions of creatures.
Predators and prey roam beneath as well as above the forest floor. Furthermore, those upstairs and
downstairs forest denizens live closed <b>linked</b> lives. Soil-dwelling bacteria and fungi break down dead
organic matter into molecules that above ground plants use for food. Those plants as well as animals,
mature and die, leaving more organic matter to fuel the folks downstairs.



Like a well-insulated house, the soil protects its tenants from extreme temperatures, and from rain and
snow. It also provides a <b>bulwark</b> against predators that roam the surface world. But the dense, protecting
soil also limits mobility. Soil creatures must be specially equipped in order to travel easily through their
dark, <b>constricting</b> realm. Earthworms and ants are the champion earth-movers, creating channels that
allow air and water to enter the soil. While ants travel relatively far from their nests, earthworms work
small areas, <b>reprocessing </b>vast amounts of soil into fertile "waste". In a single year, as much as 36 tons of
soil may pass through the alimentary tracts of all the earthworms living in an acre of soil.


<b>71.</b> It could be expected that ants ...


<b>A.</b> live only above ground <b>B.</b> are more mobile than earthworms


<b>C.</b> perform similar functions to fungi <b>D.</b> move more earth than earthworms


<b>72.</b> The word "<b>reprocessing</b>" in the passage is closest in meaning to ...


<b>A.</b> transporting <b>B.</b> converting <b>C.</b> arranging <b>D.</b> eliminating


<b>73.</b> The word " <b>bulwark</b>" in the passage is closest in meaning to ...


<b>A.</b> tomb <b>B.</b> gateway <b>C.</b> radar <b>D.</b> barrier


<b>74.</b> The main topic of the passage is ...


<b>A.</b> the life cycle of ants and worms <b>B.</b> life in the forest soil


<b>C.</b> a description of a forest scene <b>D.</b> the habits of the forest animals.


<b>75.</b> It can be inferred from the passage that the forest soil is ...



<b>A.</b> unknown to man <b>B.</b> densely inhabited <b>C.</b> devoid of life <b>D.</b> sparely inhabited


<b>76.</b> The word "<b>bustles</b>" in the passage is closest in meaning to ...


<b>A.</b> continues <b>B.</b> waits <b>C.</b> threatens <b>D.</b> is very active


<b>77.</b> The word "<b>constricting</b>" in the passage is closest in meaning to ...


<b>A.</b> limiting <b>B.</b> damp <b>C.</b> deep <b>D.</b> heavy


<b>78.</b> The soil offers creatures that live underground protection EXPECT ...


<b>A.</b> bad weather <b>B.</b> enemies <b>C.</b> extreme heat and cold <b>D.</b> bacteria and fungi


<b>79.</b> The word "<b>linked</b>" in the passage is closest in meaning to ...


<b>A.</b> chained <b>B.</b> measured <b>C.</b> related <b>D.</b> mechanical


<b>80.</b> According to the passage, what is the main function of bacteria and fungi?


<b>A.</b> to provide food for plant life <b>B.</b> to build walls in the soil


</div>
<span class='text_page_counter'>(24)</span><div class='page_container' data-page=24>

<i>Đáp án đề: 124</i>


<b> 01. </b>- - } - <b> 21. </b>- - - ~ <b> 41. </b>- | - - <b> 61. </b>- | - -


<b> 02. </b>- - } - <b> 22. </b>- | - - <b> 42. </b>- - - ~ <b> 62. </b>- - - ~


<b> 03. </b>- - } - <b> 23. </b>- - - ~ <b> 43. </b>- - - ~ <b> 63. </b>- - } -



<b> 04. </b>{ - - - <b> 24. </b>- - } - <b> 44. </b>{ - - - <b> 64. </b>- - - ~


<b> 05. </b>{ - - - <b> 25. </b>- - - ~ <b> 45. </b>- | - - <b> 65. </b>- - } -


<b> 06. </b>- - } - <b> 26. </b>- | - - <b> 46. </b>{ - - - <b> 66. </b>- - - ~


<b> 07. </b>{ - - - <b> 27. </b>- | - - <b> 47. </b>- - - ~ <b> 67. </b>{ - - -


<b> 08. </b>- - - ~ <b> 28. </b>- | - - <b> 48. </b>- - - ~ <b> 68. </b>{ - - -


<b> 09. </b>- - } - <b> 29. </b>- - } - <b> 49. </b>- - } - <b> 69. </b>- | - -


<b> 10. </b>- - } - <b> 30. </b>{ - - - <b> 50. </b>- - - ~ <b> 70. </b>{ - - -


<b> 11. </b>{ - - - <b> 31. </b>{ - - - <b> 51. </b>- - } - <b> 71. </b>- | - -


<b> 12. </b>- - } - <b> 32. </b>- - } - <b> 52. </b>- - - ~ <b> 72. </b>- | - -


<b> 13. </b>{ - - - <b> 33. </b>- - - ~ <b> 53. </b>- - } - <b> 73. </b>- - - ~


<b> 14. </b>- | - - <b> 34. </b>- | - - <b> 54. </b>- | - - <b> 74. </b>- | - -


<b> 15. </b>- - - ~ <b> 35. </b>- - } - <b> 55. </b>{ - - - <b> 75. </b>- | - -


<b> 16. </b>{ - - - <b> 36. </b>{ - - - <b> 56. </b>{ - - - <b> 76. </b>- - - ~


<b> 17. </b>- - } - <b> 37. </b>- - } - <b> 57. </b>- - } - <b> 77. </b>{ - - -


<b> 18. </b>- - } - <b> 38. </b>- - - ~ <b> 58. </b>- - - ~ <b> 78. </b>- - - ~



<b> 19. </b>{ - - - <b> 39. </b>{ - - - <b> 59. </b>- | - - <b> 79. </b>- - } -


</div>
<span class='text_page_counter'>(25)</span><div class='page_container' data-page=25>

TRƯỜNG THPT LÊ HỒNG PHONG <b> KỲ THI THỬ ĐẠI HỌC - LẦN II</b>
<b> Thị xã Bỉm Sơn </b> <b> NĂM HỌC 2010-2011</b>


<b> </b> <b> </b> <b> Môn thi: Tiếng Anh</b>


<b> Thời gian làm bài: 90 phút</b>
<b> ĐỀ CHÍNH THỨC</b>


<b> (đề thi gồm 5 trang)</b> <b> Mã đề: 125</b>
<b> </b>


Họ và tên thí sinh:... Số báo danh:...


<i><b>I. Mark the letter A, B, C, or D on your answer sheet to indicate the correct answer to each of the </b></i>
<i><b>following sentences</b></i>


<b>01.</b> I'd sooner ...deliver the new furniture tomorrow.


<b>A.</b> mustn't <b>B.</b> didn't <b>C.</b> wouldn't <b>D.</b> shouldn't


<b>02.</b> ... will remain the foremost world language is considered inevitable by many people.


<b>A.</b> English <b>B.</b> That English <b>C.</b> It is English that <b>D.</b> Because English


<b>03.</b> Technological advances aid in teaching, ... the basic role of teachers stays the same.


<b>A.</b> because of <b>B.</b> despite <b>C.</b> but <b>D.</b> with



<b>04.</b> As the President was absent, I was ask to ... the meeting.


<b>A.</b> govern <b>B.</b> officiate <b>C.</b> regulate <b>D.</b> chair


<b>05.</b> Only when a great deal of more information has been obtained, ...to plan a manned trip to the
planet.


<b>A.</b> will it possible <b>B.</b> it will be likely <b>C.</b> it will be possible <b>D.</b> it will probably be


<b>06.</b> How exactly did you set ...training the horses to work so well together?


<b>A.</b> empty <b>B.</b> loose <b>C.</b> up <b>D.</b> to


<b>07.</b> Scientists are still uncertain of ... originated millions of years ago.


<b>A.</b> how the universe <b>B.</b> with the universe <b>C.</b> which the universe <b>D.</b> about the universe


<b>08.</b> " What's the biggest problem in your country?" - " We have ...of energy."


<b>A.</b> a shortage <b>B.</b> shortage <b>C.</b> some shortage <b>D.</b> the shortage


<b>09.</b> ...the public's concern about the local environment this new road scheme will have to be
abandoned.


<b>A.</b> As regards <b>B.</b> In the event of <b>C.</b> However much <b>D.</b> In view of


<b>10.</b> Automobile ... propane gas emits fewer dangerous pollutants into the atmosphere.


<b>A.</b> using <b>B.</b> that are used <b>C.</b> use <b>D.</b> can use



<b>11.</b> All the cereal grains ... grow on the prairies and plains of the United States.


<b>A.</b> except the rice <b>B.</b> but rice <b>C.</b> but for rice <b>D.</b> excepting rice


<b>12.</b> A child in the first grade tends to be ... all of the other children in the class.


<b>A.</b> the same age as <b>B.</b> the same age than <b>C.</b> as old like <b>D.</b> the same old to


<b>13.</b> There's a rumour that the Nation Bank is going to ... the company I work for.


<b>A.</b> overtake <b>B.</b> take over <b>C.</b> take on <b>D.</b> take off


<b>14.</b> A dog ...on his owner's lap may refuse to eat from a bowl on the floor.


<b>A.</b> to feed <b>B.</b> fed <b>C.</b> was fed <b>D.</b> is fed


<b>15.</b> That popular magazine has many ...


<b>A.</b> subscribers <b>B.</b> spectators <b>C.</b> denominations <b>D.</b> prescriptions


<b>16.</b> When another person sneezes, you say "..."


<b>A.</b> Great you <b>B.</b> Pardon? <b>C.</b> See you <b>D.</b> Bless you


<b>17.</b> "Who broke the window?" - "..."


<b>A.</b> I am <b>B.</b> I did. Sorry. <b>C.</b> I was, thanks <b>D.</b> I did. Excuse me.


<b>18.</b> I'd like to have a bath, but there doesn't seem to be a ... or anything to stop.



<b>A.</b> plug <b>B.</b> lid <b>C.</b> cork <b>D.</b> cap


<b>19.</b> By the year 2012, many people currently employed ... their jobs.


<b>A.</b> will have lost <b>B.</b> have lost <b>C.</b> will be losing <b>D.</b> are losing


</div>
<span class='text_page_counter'>(26)</span><div class='page_container' data-page=26>

<b>A.</b> I wouldn't <b>B.</b> I don't <b>C.</b> I won't <b>D.</b> I can't


<b>21.</b> I had to get up early, ... I'd have missed the train.


<b>A.</b> otherwise <b>B.</b> yet <b>C.</b> if not <b>D.</b> but


<b>22.</b> Bigamy is a situation in which a man ... two women at the same time.


<b>A.</b> is marry to <b>B.</b> married <b>C.</b> marries to <b>D.</b> is married to


<b>23.</b> He tried to explain how he felt, but he was unable to ...his true feelings.


<b>A.</b> evolve <b>B.</b> inflict <b>C.</b> inhibit <b>D.</b> articulate


<b>24.</b> The product was withdrawn from sale because there was no longer any ... for it.


<b>A.</b> claim <b>B.</b> call <b>C.</b> order <b>D.</b> interest


<b>25.</b> - Janet: "Do you feel like going to the cinema tonight?" - Susan: "..."


<b>A.</b> I don't agree, I am afraid. <b>B.</b> I feel very bored.


<b>C.</b> That would be great <b>D.</b> You're welcome



<b>26.</b> "Can I use your phone?" - "..."


<b>A.</b> Of course <b>B.</b> You must <b>C.</b> You may not <b>D.</b> No, you can't


<b>27.</b> No one can visit her because she has a very ...disease.


<b>A.</b> intoxicating <b>B.</b> corporal <b>C.</b> contagious <b>D.</b> exasperating


<b>28.</b> She should have been here but she's ... flu.


<b>A.</b> gone down with <b>B.</b> come in for <b>C.</b> gone through with <b>D.</b> come up against


<b>29.</b> This film ... several scenes which are very funny.


<b>A.</b> depicts <b>B.</b> pictures <b>C.</b> illustrates <b>D.</b> features


<b>30.</b> Their climate is not dry at all; in fact, they have ...of water.


<b>A.</b> an abundance <b>B.</b> a conglomeration <b>C.</b> a redundancy <b>D.</b> a deficiency


<i><b>II. Mark the letter A, B, C, or D on your answer sheet to indicate the word that differs from the rest in </b></i>
<i><b>the position of the main stress in each of the following sentences.</b></i>


<b>31.A.</b> competitive <b>B.</b> pedestrian <b>C.</b> catalogue <b>D.</b> experienced


<b>32.A.</b> envelope <b>B.</b> insurance <b>C.</b> government <b>D.</b> lavatory


<b>33.A.</b> fortunately <b>B.</b> imprudent <b>C.</b> preferably <b>D.</b> screwdriver



<b>34.A.</b> nuclear <b>B.</b> mistake <b>C.</b> machine <b>D.</b> refer


<b>35.A.</b> hotel <b>B.</b> cassette <b>C.</b> police <b>D.</b> injured


<i><b>III. Read the following passage and mark the letter A, B, C or D on your answer sheet to indicate the </b></i>
<i><b>correct answer to each of the following questions.</b></i>


Lead poisoning in children is a major health concern. Both low and high doses of lead can have serious
effects. Children <b>exposed to</b> high doses of lead often <b>suffer</b> permanent nerve damage, mental retardation,
short attention spans, distractibility, poor academic performance, and behavioural problems. This is not a
new concern. As early as 1904, lead poisoning in children was linked to lead-based paint. Microscopic lead
particles from paint are absorbed into bloodstream when the children ingest flakes of <b>chipped</b> paint, plaster
or paint dust from suckling, or chewing on toys and other objects painted with lead-based paint. Despite
amount of lead used in their paint, this source of leading poisoning is still the most common and most
dangerous. Children living in older, <b>dilapidated</b> houses are particularly <b>at risk</b>.


<b>36.</b> What is the main topic of the passage?


<b>A.</b> Major health concern for children. <b>B.</b> Lead poisoning in children.


<b>C.</b> Problems with household paint. <b>D.</b> Lead paint in older houses.


<b>37.</b> What does the author imply in the final sentence of the passage?


<b>A.</b> Poor people did not comply with the regulations.


<b>B.</b> Lead-based paint chips off more easily than newer paints.


<b>C.</b> Old homes need to be rebuilt in order to be safe for children.



<b>D.</b> Old homes were painted with lead-based paint.


<b>38.</b> Which of the following is closest in meaning to "<b>chipped</b>"?


<b>A.</b> unhealthy <b>B.</b> sprayed <b>C.</b> canned <b>D.</b> fragmented


</div>
<span class='text_page_counter'>(27)</span><div class='page_container' data-page=27>

<b>A.</b> Paint companies have always followed regulations regarding the amount of lead used in their paint.


<b>B.</b> Paint companies can no longer use lead in their paint.


<b>C.</b> Paint companies aren't required to limit the amount lead used in their paint.


<b>D.</b> Paint companies must limit the amount lead used in their paint.


<b>40.</b> According to the passage, what is the most common source of lead poisoning in children?


<b>A.</b> dilapidated houses <b>B.</b> painted toys <b>C.</b> household dust <b>D.</b> lead-based paint


<b>41.</b> Which of the following is closest in meaning to the phrase "<b>at risk</b>" ?


<b>A.</b> no harm <b>B.</b> in danger <b>C.</b> in safe way <b>D.</b> in good condition


<b>42.</b> Which of the following is NOT true?


<b>A.</b> Lead poisoning can lead to mental retardation.


<b>B.</b> Lead poisoning in children was linked to lead-based paint in the 1900s.


<b>C.</b> Only high dose of lead can have serious effects.



<b>D.</b> American paint companies today must comply with strict regulations regarding the amount of lead
used in their paint.


<b>43.</b> Which of the following is closest in meaning to the word "<b>suffer</b>" ?.


<b>A.</b> grieve with <b>B.</b> feel paint from <b>C.</b> experience <b>D.</b> reveal


<b>44.</b> The phrase "<b>exposed to</b>" could be best replaced by which of the following?


<b>A.</b> familiar with <b>B.</b> in contact with <b>C.</b> conducting <b>D.</b> displaying


<b>45.</b> The word " <b>dilapidated " </b>is closest in meaning to which of the following?


<b>A.</b> broken down <b>B.</b> fell down <b>C.</b> poorly painted <b>D.</b> unpainted


<i><b>IV. Mark the letter A, B, C or D on your answer sheet to indicate the sentence that is closest in meaning </b></i>
<i><b>to each of following sentences.</b></i>


<b>46.</b> My career as a teacher began 14 years ago.


<b>A.</b> I have been a teacher for 14 years now. <b>B.</b> I have started teaching for 14 years now.


<b>C.</b> For 14 years have I been a teacher. <b>D.</b> I was a teacher for 14 years.


<b>47.</b> "How beautiful is the dress you have just bought!" Peter said to Mary.


<b>A.</b> Peter said thanks to Mary for her beautiful dress


<b>B.</b> Peter asked Mary how she had just bought her beautiful dress.



<b>C.</b> Peter promised to buy Mary a beautiful dress.


<b>D.</b> Peter complimented Mary on her beautiful dress.


<b>48.</b> This affair does not concern you.


<b>A.</b> Don't do this affair. <b>B.</b> Your concern is to do this affair.


<b>C.</b> This affair is no business of yours. <b>D.</b> This affair is not interesting.


<b>49.</b> My father finds maps hard to follow.


<b>A.</b> Map-reading is not interesting to my father at all. <b>B.</b> My father always gets lost.


<b>C.</b> My father has trouble following maps. <b>D.</b> My father can't read maps at all.


<b>50.</b> Not many people attended the meeting.


<b>A.</b> There was a poor turn-out for the meeting. <b>B.</b> Too many people turned out at the meeting.


<b>C.</b> People didn't show up at the meeting. <b>D.</b> No people came to the meeting


<b>51.</b> My father regretted working for that company.


<b>A.</b> If only my father was not working for that company.


<b>B.</b> It is my father's regret working for that company.


<b>C.</b> My father didn't like working for that company.



<b>D.</b> My father wished he hadn't worked for that company.


<b>52.</b> She started work three months ago.


<b>A.</b> She had been working for three months. <b>B.</b> She is working here for three months now.


<b>C.</b> It is three months since she started work. <b>D.</b> It's three months that she worked for.


<b>53.</b> We came home early to avoid the coming storm.


<b>A.</b> Because it was predicted storming, we decided to come home earlier than unusual.


<b>B.</b> In order to avoiding the coming storm, we came home early.


</div>
<span class='text_page_counter'>(28)</span><div class='page_container' data-page=28>

<b>D.</b> We didn't come home late to not get the coming storm.


<b>54.</b> This is the most delicious meat I've ever eaten.


<b>A.</b> At no time in my life have I never tasted this excellent meat.


<b>B.</b> I have never eaten such more delicious meat.


<b>C.</b> Not ever in my life have I had such a wonderful meat.


<b>D.</b> Never in my life have I had such a delicious meat


<b>55.</b> She has lost her appetite recently.


<b>A.</b> She has eaten a lot of food recently. <b>B.</b> She has gone off food recently.



<b>C.</b> Her appetite has been very good. <b>D.</b> She hasn't had any food recently.


<i><b>V. Read the following passage and mark the letter A, B, C or D on your answer sheet to indicate the </b></i>
<i><b>correct word for each of the blanks.</b></i>


In this age of ...(56)... telephone networks and electronic mail, it seems that fewer and even fewer
people are taking time to sit down and write letters ...(57)... friends and relatives. For hundreds of years,
letters were the only way to keep ...(58)... people who were any distance away and letter-writing was seen
as an important skill for all learned people...(59).... Gradually, ...(60)..., the importance of writing letters is
decreasing to a point that majority of us have to ...(61)... a special effort to turn out something worthwhile
when we apply for a job or make an appointment. In business circles the tendency is for routine
communications to become shorter. ...(62)...clients may appreciate a detailed letter, an employee who
sends out long letter is often regarded as ...(63).... Many people prefer the telephone in all circumstances
and its speed is essential in many situations but ...(64)... have you put the telephone down, dissatisfied
with what you have managed to say? I don't think I'll throw my ...(65)... away yet.


<b>56.A.</b> all are correct <b>B.</b> highly-developed <b>C.</b> advanced <b>D.</b> progressive


<b>57.A.</b> from <b>B.</b> with <b>C.</b> for <b>D.</b> to


<b>58.A.</b> in step with <b>B.</b> on good terms with <b>C.</b> in favour of <b>D.</b> in contact with


<b>59.A.</b> mastering <b>B.</b> to be mastered <b>C.</b> to master <b>D.</b> mastered


<b>60.A.</b> in short <b>B.</b> however <b>C.</b> therefore <b>D.</b> for example


<b>61.A.</b> create <b>B.</b> do <b>C.</b> cause <b>D.</b> make


<b>62.A.</b> As though <b>B.</b> Despite <b>C.</b> Even though <b>D.</b> However



<b>63.A.</b> unimportant <b>B.</b> impossible <b>C.</b> unusual <b>D.</b> inefficient


<b>64.A.</b> how often <b>B.</b> how about <b>C.</b> how long <b>D.</b> how much


<b>65.A.</b> letter <b>B.</b> telephone <b>C.</b> pen <b>D.</b> effort


<i><b>VI. Mark the letter A, B, C, or D on your answer sheet to show the underlined part that needs </b></i>
<i><b>correction. </b></i>


<b>66.</b> Vitamin K providing the necessary impetus for the synthesis of at least two proteins involved in
blood clotting.


<b>A.</b> necessary <b>B.</b> providing <b>C.</b> at least <b>D.</b> involved


<b>67.</b> Organisms and their cells live by maintaining a constant exchange of elemental, ions, minerals and
gases.


<b>A.</b> constant <b>B.</b> of elemental <b>C.</b> gases <b>D.</b> by maintaining


<b>68.</b> Perhaps the most unique thing about carbon atoms are their ability to combine with themselves.


<b>A.</b> about <b>B.</b> to combine <b>C.</b> the most <b>D.</b> are


<b>69.</b> For the past few years, researchers have perfecting their control over the movements of cells and
microbes by using low power laser beams.


<b>A.</b> their <b>B.</b> using <b>C.</b> few years <b>D.</b> have perfecting


<b>70.</b> The nests of most bird species are strategic placed to camouflage them against predators.



</div>
<span class='text_page_counter'>(29)</span><div class='page_container' data-page=29>

<i><b>VII. Read the following passage and mark the letter A, B, C or D on your answer sheet to indicate the </b></i>
<i><b>correct answer to each of the following questions.</b></i>


Watching for wildlife in the forests, we rarely see past the surface of things. Standing on the ground
floor, we scan the leafy rafters, entirely overlooking the living world in the soil beneath our feet. The
forest's basement is a secret world. As different from our own world as water is from air, the soil seems
quiet, even dead. But life <b>bustles</b> down below: a cubic inch of top soil may contain billions of creatures.
Predators and prey roam beneath as well as above the forest floor. Furthermore, those upstairs and
downstairs forest denizens live closed <b>linked</b> lives. Soil-dwelling bacteria and fungi break down dead
organic matter into molecules that above ground plants use for food. Those plants as well as animals,
mature and die, leaving more organic matter to fuel the folks downstairs.


Like a well-insulated house, the soil protects its tenants from extreme temperatures, and from rain and
snow. It also provides a <b>bulwark</b> against predators that roam the surface world. But the dense, protecting
soil also limits mobility. Soil creatures must be specially equipped in order to travel easily through their
dark, <b>constricting</b> realm. Earthworms and ants are the champion earth-movers, creating channels that
allow air and water to enter the soil. While ants travel relatively far from their nests, earthworms work
small areas, <b>reprocessing </b>vast amounts of soil into fertile "waste". In a single year, as much as 36 tons of
soil may pass through the alimentary tracts of all the earthworms living in an acre of soil.


<b>71.</b> It can be inferred from the passage that the forest soil is ...


<b>A.</b> unknown to man <b>B.</b> devoid of life <b>C.</b> densely inhabited <b>D.</b> sparely inhabited


<b>72.</b> The word "<b>constricting</b>" in the passage is closest in meaning to ...


<b>A.</b> damp <b>B.</b> heavy <b>C.</b> limiting <b>D.</b> deep


<b>73.</b> The word "<b>linked</b>" in the passage is closest in meaning to ...



<b>A.</b> chained <b>B.</b> measured <b>C.</b> related <b>D.</b> mechanical


<b>74.</b> The soil offers creatures that live underground protection EXPECT ...


<b>A.</b> bad weather <b>B.</b> enemies <b>C.</b> extreme heat and cold <b>D.</b> bacteria and fungi


<b>75.</b> The word "<b>bustles</b>" in the passage is closest in meaning to ...


<b>A.</b> is very active <b>B.</b> waits <b>C.</b> continues <b>D.</b> threatens


<b>76.</b> According to the passage, what is the main function of bacteria and fungi?


<b>A.</b> to build walls in the soil <b>B.</b> to provide food for plant life


<b>C.</b> to help aerate the soil <b>D.</b> to kill mature plants


<b>77.</b> It could be expected that ants ...


<b>A.</b> live only above ground <b>B.</b> perform similar functions to fungi


<b>C.</b> are more mobile than earthworms <b>D.</b> move more earth than earthworms


<b>78.</b> The main topic of the passage is ...


<b>A.</b> life in the forest soil <b>B.</b> the habits of the forest animals.


<b>C.</b> the life cycle of ants and worms <b>D.</b> a description of a forest scene


<b>79.</b> The word " <b>bulwark</b>" in the passage is closest in meaning to ...



<b>A.</b> radar <b>B.</b> tomb <b>C.</b> barrier <b>D.</b> gateway


<b>80.</b> The word "<b>reprocessing</b>" in the passage is closest in meaning to ...


</div>
<span class='text_page_counter'>(30)</span><div class='page_container' data-page=30>

<i>Đáp án đề: 125</i>


<b> 01. </b>- - - ~ <b> 21. </b>{ - - - <b> 41. </b>- | - - <b> 61. </b>- - - ~


<b> 02. </b>- | - - <b> 22. </b>- - - ~ <b> 42. </b>- - } - <b> 62. </b>- - } -


<b> 03. </b>- - } - <b> 23. </b>- - - ~ <b> 43. </b>- - } - <b> 63. </b>- - - ~


<b> 04. </b>- - - ~ <b> 24. </b>- | - - <b> 44. </b>- | - - <b> 64. </b>{ - - -


<b> 05. </b>{ - - - <b> 25. </b>- - } - <b> 45. </b>{ - - - <b> 65. </b>- - } -


<b> 06. </b>- - } - <b> 26. </b>{ - - - <b> 46. </b>{ - - - <b> 66. </b>- | - -


<b> 07. </b>{ - - - <b> 27. </b>- - } - <b> 47. </b>- - - ~ <b> 67. </b>- | - -


<b> 08. </b>{ - - - <b> 28. </b>{ - - - <b> 48. </b>- - } - <b> 68. </b>- - - ~


<b> 09. </b>- - - ~ <b> 29. </b>- - - ~ <b> 49. </b>- - } - <b> 69. </b>- - - ~


<b> 10. </b>{ - - - <b> 30. </b>{ - - - <b> 50. </b>{ - - - <b> 70. </b>- - - ~


<b> 11. </b>- | - - <b> 31. </b>- - } - <b> 51. </b>- - - ~ <b> 71. </b>- - } -


<b> 12. </b>{ - - - <b> 32. </b>- | - - <b> 52. </b>- - } - <b> 72. </b>- - } -



<b> 13. </b>- | - - <b> 33. </b>- | - - <b> 53. </b>- - } - <b> 73. </b>- - } -


<b> 14. </b>- | - - <b> 34. </b>{ - - - <b> 54. </b>- | - - <b> 74. </b>- - - ~


<b> 15. </b>{ - - - <b> 35. </b>{ - - - <b> 55. </b>- | - - <b> 75. </b>{ - - -


<b> 16. </b>- - - ~ <b> 36. </b>- | - - <b> 56. </b>{ - - - <b> 76. </b>- | - -


<b> 17. </b>- | - - <b> 37. </b>- - } - <b> 57. </b>- - - ~ <b> 77. </b>- - } -


<b> 18. </b>{ - - - <b> 38. </b>- - - ~ <b> 58. </b>- - - ~ <b> 78. </b>{ - - -


<b> 19. </b>{ - - - <b> 39. </b>- - - ~ <b> 59. </b>- - } - <b> 79. </b>- - } -


</div>
<span class='text_page_counter'>(31)</span><div class='page_container' data-page=31>

TRƯỜNG THPT LÊ HỒNG PHONG <b> KỲ THI THỬ ĐẠI HỌC - LẦN II</b>
<b> Thị xã Bỉm Sơn </b> <b> NĂM HỌC 2010-2011</b>


<b> </b> <b> </b> <b> Môn thi: Tiếng Anh</b>


<b> Thời gian làm bài: 90 phút</b>
<b> ĐỀ CHÍNH THỨC</b>


<b> (đề thi gồm 5 trang)</b> <b>Mã đề: 12</b>
<b> </b>


Họ và tên thí sinh:... Số báo danh:...


<i><b>I. Mark the letter A, B, C, or D on your answer sheet to indicate the word that differs from the rest in </b></i>
<i><b>the position of the main stress in each of the following sentences.</b></i>



<b>01.A.</b> preferably <b>B.</b> screwdriver <b>C.</b> fortunately <b>D.</b> imprudent


<b>02.A.</b> competitive <b>B.</b> pedestrian <b>C.</b> experienced <b>D.</b> catalogue


<b>03.A.</b> cassette <b>B.</b> injured <b>C.</b> hotel <b>D.</b> police


<b>04.A.</b> lavatory <b>B.</b> insurance <b>C.</b> government <b>D.</b> envelope


<b>05.A.</b> nuclear <b>B.</b> machine <b>C.</b> refer <b>D.</b> mistake


<i><b>II. Mark the letter A, B, C or D on your answer sheet to indicate the sentence that is closest in meaning </b></i>
<i><b>to each of following sentences.</b></i>


<b>06.</b> My father regretted working for that company.


<b>A.</b> It is my father's regret working for that company.


<b>B.</b> My father wished he hadn't worked for that company.


<b>C.</b> My father didn't like working for that company.


<b>D.</b> If only my father was not working for that company.


<b>07.</b> My career as a teacher began 14 years ago.


<b>A.</b> I was a teacher for 14 years. <b>B.</b> I have started teaching for 14 years now.


<b>C.</b> For 14 years have I been a teacher. <b>D.</b> I have been a teacher for 14 years now.


<b>08.</b> We came home early to avoid the coming storm.



<b>A.</b> In order to avoiding the coming storm, we came home early.


<b>B.</b> We came home early in order to avoid the coming storm.


<b>C.</b> Because it was predicted storming, we decided to come home earlier than unusual.


<b>D.</b> We didn't come home late to not get the coming storm.


<b>09.</b> Not many people attended the meeting.


<b>A.</b> No people came to the meeting <b>B.</b> There was a poor turn-out for the meeting.


<b>C.</b> Too many people turned out at the meeting. <b>D.</b> People didn't show up at the meeting.


<b>10.</b> This affair does not concern you.


<b>A.</b> Don't do this affair. <b>B.</b> This affair is no business of yours.


<b>C.</b> This affair is not interesting. <b>D.</b> Your concern is to do this affair.


<b>11.</b> "How beautiful is the dress you have just bought!" Peter said to Mary.


<b>A.</b> Peter said thanks to Mary for her beautiful dress


<b>B.</b> Peter asked Mary how she had just bought her beautiful dress.


<b>C.</b> Peter complimented Mary on her beautiful dress.


<b>D.</b> Peter promised to buy Mary a beautiful dress.



<b>12.</b> My father finds maps hard to follow.


<b>A.</b> Map-reading is not interesting to my father at all. <b>B.</b> My father always gets lost.


<b>C.</b> My father can't read maps at all. <b>D.</b> My father has trouble following maps.


<b>13.</b> She has lost her appetite recently.


<b>A.</b> She has eaten a lot of food recently. <b>B.</b> She hasn't had any food recently.


<b>C.</b> She has gone off food recently. <b>D.</b> Her appetite has been very good.


<b>14.</b> She started work three months ago.


<b>A.</b> It's three months that she worked for. <b>B.</b> It is three months since she started work.


</div>
<span class='text_page_counter'>(32)</span><div class='page_container' data-page=32>

<b>15.</b> This is the most delicious meat I've ever eaten.


<b>A.</b> Never in my life have I had such a delicious meat


<b>B.</b> At no time in my life have I never tasted this excellent meat.


<b>C.</b> Not ever in my life have I had such a wonderful meat.


<b>D.</b> I have never eaten such more delicious meat.


<i><b>III. Read the following passage and mark the letter A, B, C or D on your answer sheet to indicate the </b></i>
<i><b>correct word for each of the blanks.</b></i>



In this age of ...(16)... telephone networks and electronic mail, it seems that fewer and even fewer
people are taking time to sit down and write letters ...(17)... friends and relatives. For hundreds of years,
letters were the only way to keep ...(18)... people who were any distance away and letter-writing was seen
as an important skill for all learned people...(19).... Gradually, ...(20)..., the importance of writing letters is
decreasing to a point that majority of us have to ...(21)... a special effort to turn out something worthwhile
when we apply for a job or make an appointment. In business circles the tendency is for routine
communications to become shorter. ...(22)...clients may appreciate a detailed letter, an employee who
sends out long letter is often regarded as ...(23).... Many people prefer the telephone in all circumstances
and its speed is essential in many situations but ...(24)... have you put the telephone down, dissatisfied
with what you have managed to say? I don't think I'll throw my ...(25)... away yet.


<b>16.A.</b> advanced <b>B.</b> all are correct <b>C.</b> progressive <b>D.</b> highly-developed


<b>17.A.</b> from <b>B.</b> to <b>C.</b> for <b>D.</b> with


<b>18.A.</b> in contact with <b>B.</b> in step with <b>C.</b> on good terms with <b>D.</b> in favour of


<b>19.A.</b> to be mastered <b>B.</b> mastering <b>C.</b> mastered <b>D.</b> to master


<b>20.A.</b> for example <b>B.</b> in short <b>C.</b> however <b>D.</b> therefore


<b>21.A.</b> make <b>B.</b> cause <b>C.</b> create <b>D.</b> do


<b>22.A.</b> However <b>B.</b> As though <b>C.</b> Despite <b>D.</b> Even though


<b>23.A.</b> inefficient <b>B.</b> unimportant <b>C.</b> impossible <b>D.</b> unusual


<b>24.A.</b> how long <b>B.</b> how often <b>C.</b> how about <b>D.</b> how much


<b>25.A.</b> pen <b>B.</b> letter <b>C.</b> telephone <b>D.</b> effort



<i><b>IV. Mark the letter A, B, C, or D on your answer sheet to indicate the correct answer to each of the </b></i>
<i><b>following sentences</b></i>


<b>26.</b> I'd sooner ...deliver the new furniture tomorrow.


<b>A.</b> wouldn't <b>B.</b> didn't <b>C.</b> mustn't <b>D.</b> shouldn't


<b>27.</b> A child in the first grade tends to be ... all of the other children in the class.


<b>A.</b> the same old to <b>B.</b> as old like <b>C.</b> the same age than <b>D.</b> the same age as


<b>28.</b> She should have been here but she's ... flu.


<b>A.</b> gone through with <b>B.</b> come in for <b>C.</b> gone down with <b>D.</b> come up against


<b>29.</b> Only when a great deal of more information has been obtained, ...to plan a manned trip to the
planet.


<b>A.</b> will it possible <b>B.</b> it will be possible <b>C.</b> it will be likely <b>D.</b> it will probably be


<b>30.</b> He tried to explain how he felt, but he was unable to ...his true feelings.


<b>A.</b> articulate <b>B.</b> inhibit <b>C.</b> evolve <b>D.</b> inflict


<b>31.</b> No one can visit her because she has a very ...disease.


<b>A.</b> intoxicating <b>B.</b> exasperating <b>C.</b> contagious <b>D.</b> corporal


<b>32.</b> ... will remain the foremost world language is considered inevitable by many people.



<b>A.</b> It is English that <b>B.</b> English <b>C.</b> That English <b>D.</b> Because English


<b>33.</b> ...the public's concern about the local environment this new road scheme will have to be
abandoned.


<b>A.</b> In the event of <b>B.</b> However much <b>C.</b> In view of <b>D.</b> As regards


<b>34.</b> By the year 2012, many people currently employed ... their jobs.


<b>A.</b> are losing <b>B.</b> will have lost <b>C.</b> have lost <b>D.</b> will be losing


</div>
<span class='text_page_counter'>(33)</span><div class='page_container' data-page=33>

<b>A.</b> I did. Sorry. <b>B.</b> I am <b>C.</b> I was, thanks <b>D.</b> I did. Excuse me.


<b>36.</b> I'd like to have a bath, but there doesn't seem to be a ... or anything to stop.


<b>A.</b> plug <b>B.</b> cap <b>C.</b> cork <b>D.</b> lid


<b>37.</b> All the cereal grains ... grow on the prairies and plains of the United States.


<b>A.</b> but rice <b>B.</b> excepting rice <b>C.</b> but for rice <b>D.</b> except the rice


<b>38.</b> This film ... several scenes which are very funny.


<b>A.</b> features <b>B.</b> depicts <b>C.</b> pictures <b>D.</b> illustrates


<b>39.</b> I had to get up early, ... I'd have missed the train.


<b>A.</b> yet <b>B.</b> if not <b>C.</b> otherwise <b>D.</b> but



<b>40.</b> "Don't tell anyone my new address" - "..."


<b>A.</b> I won't <b>B.</b> I wouldn't <b>C.</b> I don't <b>D.</b> I can't


<b>41.</b> " What's the biggest problem in your country?" - " We have ...of energy."


<b>A.</b> the shortage <b>B.</b> shortage <b>C.</b> some shortage <b>D.</b> a shortage


<b>42.</b> How exactly did you set ...training the horses to work so well together?


<b>A.</b> empty <b>B.</b> loose <b>C.</b> up <b>D.</b> to


<b>43.</b> "Can I use your phone?" - "..."


<b>A.</b> You may not <b>B.</b> Of course <b>C.</b> No, you can't <b>D.</b> You must


<b>44.</b> Their climate is not dry at all; in fact, they have ...of water.


<b>A.</b> an abundance <b>B.</b> a redundancy <b>C.</b> a deficiency <b>D.</b> a conglomeration


<b>45.</b> The product was withdrawn from sale because there was no longer any ... for it.


<b>A.</b> order <b>B.</b> interest <b>C.</b> call <b>D.</b> claim


<b>46.</b> Automobile ... propane gas emits fewer dangerous pollutants into the atmosphere.


<b>A.</b> can use <b>B.</b> that are used <b>C.</b> using <b>D.</b> use


<b>47.</b> There's a rumour that the Nation Bank is going to ... the company I work for.



<b>A.</b> overtake <b>B.</b> take on <b>C.</b> take off <b>D.</b> take over


<b>48.</b> Bigamy is a situation in which a man ... two women at the same time.


<b>A.</b> marries to <b>B.</b> is marry to <b>C.</b> married <b>D.</b> is married to


<b>49.</b> - Janet: "Do you feel like going to the cinema tonight?" - Susan: "..."


<b>A.</b> That would be great <b>B.</b> You're welcome


<b>C.</b> I don't agree, I am afraid. <b>D.</b> I feel very bored.


<b>50.</b> A dog ...on his owner's lap may refuse to eat from a bowl on the floor.


<b>A.</b> is fed <b>B.</b> to feed <b>C.</b> was fed <b>D.</b> fed


<b>51.</b> Technological advances aid in teaching, ... the basic role of teachers stays the same.


<b>A.</b> because of <b>B.</b> with <b>C.</b> despite <b>D.</b> but


<b>52.</b> As the President was absent, I was ask to ... the meeting.


<b>A.</b> regulate <b>B.</b> officiate <b>C.</b> govern <b>D.</b> chair


<b>53.</b> When another person sneezes, you say "..."


<b>A.</b> Pardon? <b>B.</b> Bless you <b>C.</b> Great you <b>D.</b> See you


<b>54.</b> Scientists are still uncertain of ... originated millions of years ago.



<b>A.</b> how the universe <b>B.</b> about the universe <b>C.</b> with the universe <b>D.</b> which the universe


<b>55.</b> That popular magazine has many ...


<b>A.</b> spectators <b>B.</b> denominations <b>C.</b> prescriptions <b>D.</b> subscribers


<i><b>V. Mark the letter A, B, C, or D on your answer sheet to show the underlined part that needs correction.</b></i>


<b>56.</b> The nests of most bird species are strategic placed to camouflage them against predators.


<b>A.</b> strategic <b>B.</b> species <b>C.</b> against predators. <b>D.</b> most


<b>57.</b> Organisms and their cells live by maintaining a constant exchange of elemental, ions, minerals and
gases.


<b>A.</b> by maintaining <b>B.</b> constant <b>C.</b> of elemental <b>D.</b> gases


<b>58.</b> Perhaps the most unique thing about carbon atoms are their ability to combine with themselves.


</div>
<span class='text_page_counter'>(34)</span><div class='page_container' data-page=34>

<b>59.</b> Vitamin K providing the necessary impetus for the synthesis of at least two proteins involved in
blood clotting.


<b>A.</b> at least <b>B.</b> providing <b>C.</b> involved <b>D.</b> necessary


<b>60.</b> For the past few years, researchers have perfecting their control over the movements of cells and
microbes by using low power laser beams.


<b>A.</b> using <b>B.</b> their <b>C.</b> few years <b>D.</b> have perfecting


<i><b>VI. Read the following passage and mark the letter A, B, C or D on your answer sheet to indicate the </b></i>


<i><b>correct answer to each of the following questions.</b></i>


Lead poisoning in children is a major health concern. Both low and high doses of lead can have serious
effects. Children <b>exposed to</b> high doses of lead often <b>suffer</b> permanent nerve damage, mental retardation,
short attention spans, distractibility, poor academic performance, and behavioural problems. This is not a
new concern. As early as 1904, lead poisoning in children was linked to lead-based paint. Microscopic lead
particles from paint are absorbed into bloodstream when the children ingest flakes of <b>chipped</b> paint, plaster
or paint dust from suckling, or chewing on toys and other objects painted with lead-based paint. Despite
amount of lead used in their paint, this source of leading poisoning is still the most common and most
dangerous. Children living in older, <b>dilapidated</b> houses are particularly <b>at risk</b>.


<b>61.</b> The phrase "<b>exposed to</b>" could be best replaced by which of the following?


<b>A.</b> displaying <b>B.</b> familiar with <b>C.</b> in contact with <b>D.</b> conducting


<b>62.</b> Which of the following is closest in meaning to "<b>chipped</b>"?


<b>A.</b> canned <b>B.</b> unhealthy <b>C.</b> sprayed <b>D.</b> fragmented


<b>63.</b> Which of the following is closest in meaning to the phrase "<b>at risk</b>" ?


<b>A.</b> in good condition <b>B.</b> no harm <b>C.</b> in danger <b>D.</b> in safe way


<b>64.</b> The word " <b>dilapidated " </b>is closest in meaning to which of the following?


<b>A.</b> poorly painted <b>B.</b> broken down <b>C.</b> fell down <b>D.</b> unpainted


<b>65.</b> What is the main topic of the passage?


<b>A.</b> Major health concern for children. <b>B.</b> Lead paint in older houses.



<b>C.</b> Lead poisoning in children. <b>D.</b> Problems with household paint.


<b>66.</b> Which of the following is NOT true?


<b>A.</b> Only high dose of lead can have serious effects.


<b>B.</b> Lead poisoning can lead to mental retardation.


<b>C.</b> Lead poisoning in children was linked to lead-based paint in the 1900s.


<b>D.</b> American paint companies today must comply with strict regulations regarding the amount of lead
used in their paint.


<b>67.</b> Which of the following is closest in meaning to the word "<b>suffer</b>" ?.


<b>A.</b> reveal <b>B.</b> grieve with <b>C.</b> feel paint from <b>D.</b> experience


<b>68.</b> According to the passage, what is the most common source of lead poisoning in children?


<b>A.</b> household dust <b>B.</b> painted toys <b>C.</b> lead-based paint <b>D.</b> dilapidated houses


<b>69.</b> What does the author imply in the final sentence of the passage?


<b>A.</b> Old homes need to be rebuilt in order to be safe for children.


<b>B.</b> Poor people did not comply with the regulations.


<b>C.</b> Lead-based paint chips off more easily than newer paints.



<b>D.</b> Old homes were painted with lead-based paint.


<b>70.</b> Which of the following can be inferred from the passage?


<b>A.</b> Paint companies can no longer use lead in their paint.


<b>B.</b> Paint companies aren't required to limit the amount lead used in their paint.


<b>C.</b> Paint companies have always followed regulations regarding the amount of lead used in their paint.


</div>
<span class='text_page_counter'>(35)</span><div class='page_container' data-page=35>

<i><b>VII. Read the following passage and mark the letter A, B, C or D on your answer sheet to indicate the </b></i>
<i><b>correct answer to each of the following questions.</b></i>


Watching for wildlife in the forests, we rarely see past the surface of things. Standing on the ground
floor, we scan the leafy rafters, entirely overlooking the living world in the soil beneath our feet. The
forest's basement is a secret world. As different from our own world as water is from air, the soil seems
quiet, even dead. But life <b>bustles</b> down below: a cubic inch of top soil may contain billions of creatures.
Predators and prey roam beneath as well as above the forest floor. Furthermore, those upstairs and
downstairs forest denizens live closed <b>linked</b> lives. Soil-dwelling bacteria and fungi break down dead
organic matter into molecules that above ground plants use for food. Those plants as well as animals,
mature and die, leaving more organic matter to fuel the folks downstairs.


Like a well-insulated house, the soil protects its tenants from extreme temperatures, and from rain and
snow. It also provides a <b>bulwark</b> against predators that roam the surface world. But the dense, protecting
soil also limits mobility. Soil creatures must be specially equipped in order to travel easily through their
dark, <b>constricting</b> realm. Earthworms and ants are the champion earth-movers, creating channels that
allow air and water to enter the soil. While ants travel relatively far from their nests, earthworms work
small areas, <b>reprocessing </b>vast amounts of soil into fertile "waste". In a single year, as much as 36 tons of
soil may pass through the alimentary tracts of all the earthworms living in an acre of soil.



<b>71.</b> The soil offers creatures that live underground protection EXPECT ...


<b>A.</b> extreme heat and cold <b>B.</b> enemies <b>C.</b> bacteria and fungi <b>D.</b> bad weather


<b>72.</b> It can be inferred from the passage that the forest soil is ...


<b>A.</b> sparely inhabited <b>B.</b> unknown to man <b>C.</b> densely inhabited <b>D.</b> devoid of life


<b>73.</b> According to the passage, what is the main function of bacteria and fungi?


<b>A.</b> to build walls in the soil <b>B.</b> to kill mature plants


<b>C.</b> to provide food for plant life <b>D.</b> to help aerate the soil


<b>74.</b> The word "<b>bustles</b>" in the passage is closest in meaning to ...


<b>A.</b> continues <b>B.</b> waits <b>C.</b> is very active <b>D.</b> threatens


<b>75.</b> The word "<b>constricting</b>" in the passage is closest in meaning to ...


<b>A.</b> limiting <b>B.</b> deep <b>C.</b> damp <b>D.</b> heavy


<b>76.</b> The word "<b>linked</b>" in the passage is closest in meaning to ...


<b>A.</b> related <b>B.</b> chained <b>C.</b> mechanical <b>D.</b> measured


<b>77.</b> The word " <b>bulwark</b>" in the passage is closest in meaning to ...


<b>A.</b> barrier <b>B.</b> tomb <b>C.</b> gateway <b>D.</b> radar



<b>78.</b> It could be expected that ants ...


<b>A.</b> perform similar functions to fungi <b>B.</b> move more earth than earthworms


<b>C.</b> live only above ground <b>D.</b> are more mobile than earthworms


<b>79.</b> The main topic of the passage is ...


<b>A.</b> a description of a forest scene <b>B.</b> the life cycle of ants and worms


<b>C.</b> the habits of the forest animals. <b>D.</b> life in the forest soil


<b>80.</b> The word "<b>reprocessing</b>" in the passage is closest in meaning to ...


</div>
<span class='text_page_counter'>(36)</span><div class='page_container' data-page=36>

<i>Đáp án đề: 126</i>


<b> 01. </b>- - - ~ <b> 21. </b>{ - - - <b> 41. </b>- - - ~ <b> 61. </b>- - } -


<b> 02. </b>- - - ~ <b> 22. </b>- - - ~ <b> 42. </b>- - } - <b> 62. </b>- - - ~


<b> 03. </b>- - } - <b> 23. </b>{ - - - <b> 43. </b>- | - - <b> 63. </b>- - } -


<b> 04. </b>- | - - <b> 24. </b>- | - - <b> 44. </b>{ - - - <b> 64. </b>- | - -


<b> 05. </b>{ - - - <b> 25. </b>{ - - - <b> 45. </b>- - } - <b> 65. </b>- - } -


<b> 06. </b>- | - - <b> 26. </b>- - - ~ <b> 46. </b>- - } - <b> 66. </b>{ - - -


<b> 07. </b>- - - ~ <b> 27. </b>- - - ~ <b> 47. </b>- - - ~ <b> 67. </b>- - - ~



<b> 08. </b>- | - - <b> 28. </b>- - } - <b> 48. </b>- - - ~ <b> 68. </b>- - } -


<b> 09. </b>- | - - <b> 29. </b>{ - - - <b> 49. </b>{ - - - <b> 69. </b>{ - - -


<b> 10. </b>- | - - <b> 30. </b>{ - - - <b> 50. </b>- - - ~ <b> 70. </b>- - - ~


<b> 11. </b>- - } - <b> 31. </b>- - } - <b> 51. </b>- - - ~ <b> 71. </b>- - } -


<b> 12. </b>- - - ~ <b> 32. </b>- - } - <b> 52. </b>- - - ~ <b> 72. </b>- - } -


<b> 13. </b>- - } - <b> 33. </b>- - } - <b> 53. </b>- | - - <b> 73. </b>- - } -


<b> 14. </b>- | - - <b> 34. </b>- | - - <b> 54. </b>{ - - - <b> 74. </b>- - } -


<b> 15. </b>- - - ~ <b> 35. </b>{ - - - <b> 55. </b>- - - ~ <b> 75. </b>{ - - -


<b> 16. </b>- | - - <b> 36. </b>{ - - - <b> 56. </b>{ - - - <b> 76. </b>{ - - -


<b> 17. </b>- | - - <b> 37. </b>{ - - - <b> 57. </b>- - } - <b> 77. </b>{ - - -


<b> 18. </b>{ - - - <b> 38. </b>{ - - - <b> 58. </b>- - } - <b> 78. </b>- - - ~


<b> 19. </b>- - - ~ <b> 39. </b>- - } - <b> 59. </b>- | - - <b> 79. </b>- - - ~


</div>
<span class='text_page_counter'>(37)</span><div class='page_container' data-page=37>

TRƯỜNG THPT LÊ HỒNG PHONG <b> KỲ THI THỬ ĐẠI HỌC - LẦN II</b>
<b> Thị xã Bỉm Sơn </b> <b>NĂM HỌC 2010-2011</b>


<b> </b> <b> </b> <b> Môn thi: Tiếng Anh</b>


<b> Thời gian làm bài: 90 phút</b>


<b> ĐỀ CHÍNH THỨC</b>


<b> (đề thi gồm 5 trang)</b> <b>Mã đề: 12</b>
<b> </b>


<i> Họ và tên thí sinh:... Số báo danh:...</i>


<i><b>I. Mark the letter A, B, C or D on your answer sheet to indicate the sentence that is closest in meaning </b></i>
<i><b>to each of following sentences.</b></i>


<b>01.</b> This is the most delicious meat I've ever eaten.


<b>A.</b> Not ever in my life have I had such a wonderful meat.


<b>B.</b> I have never eaten such more delicious meat.


<b>C.</b> Never in my life have I had such a delicious meat


<b>D.</b> At no time in my life have I never tasted this excellent meat.


<b>02.</b> We came home early to avoid the coming storm.


<b>A.</b> Because it was predicted storming, we decided to come home earlier than unusual.


<b>B.</b> We didn't come home late to not get the coming storm.


<b>C.</b> In order to avoiding the coming storm, we came home early.


<b>D.</b> We came home early in order to avoid the coming storm.



<b>03.</b> "How beautiful is the dress you have just bought!" Peter said to Mary.


<b>A.</b> Peter complimented Mary on her beautiful dress.


<b>B.</b> Peter said thanks to Mary for her beautiful dress


<b>C.</b> Peter asked Mary how she had just bought her beautiful dress.


<b>D.</b> Peter promised to buy Mary a beautiful dress.


<b>04.</b> Not many people attended the meeting.


<b>A.</b> There was a poor turn-out for the meeting. <b>B.</b> Too many people turned out at the meeting.


<b>C.</b> No people came to the meeting <b>D.</b> People didn't show up at the meeting.


<b>05.</b> My father finds maps hard to follow.


<b>A.</b> My father always gets lost. <b>B.</b> My father can't read maps at all.


<b>C.</b> My father has trouble following maps. <b>D.</b> Map-reading is not interesting to my father at all.


<b>06.</b> This affair does not concern you.


<b>A.</b> Don't do this affair. <b>B.</b> This affair is no business of yours.


<b>C.</b> This affair is not interesting. <b>D.</b> Your concern is to do this affair.


<b>07.</b> My father regretted working for that company.



<b>A.</b> My father didn't like working for that company.


<b>B.</b> It is my father's regret working for that company.


<b>C.</b> If only my father was not working for that company.


<b>D.</b> My father wished he hadn't worked for that company.


<b>08.</b> My career as a teacher began 14 years ago.


<b>A.</b> For 14 years have I been a teacher. <b>B.</b> I have started teaching for 14 years now.


<b>C.</b> I have been a teacher for 14 years now. <b>D.</b> I was a teacher for 14 years.


<b>09.</b> She started work three months ago.


<b>A.</b> It's three months that she worked for. <b>B.</b> She had been working for three months.


<b>C.</b> She is working here for three months now. <b>D.</b> It is three months since she started work.


<b>10.</b> She has lost her appetite recently.


<b>A.</b> She has gone off food recently. <b>B.</b> Her appetite has been very good.


<b>C.</b> She has eaten a lot of food recently. <b>D.</b> She hasn't had any food recently.


</div>
<span class='text_page_counter'>(38)</span><div class='page_container' data-page=38>

<b>11.</b> That popular magazine has many ...


<b>A.</b> spectators <b>B.</b> denominations <b>C.</b> prescriptions <b>D.</b> subscribers



<b>12.</b> Only when a great deal of more information has been obtained, ...to plan a manned trip to the
planet.


<b>A.</b> it will be possible <b>B.</b> will it possible <b>C.</b> it will probably be <b>D.</b> it will be likely


<b>13.</b> A child in the first grade tends to be ... all of the other children in the class.


<b>A.</b> the same age than <b>B.</b> the same old to <b>C.</b> as old like <b>D.</b> the same age as


<b>14.</b> Their climate is not dry at all; in fact, they have ...of water.


<b>A.</b> a conglomeration <b>B.</b> a redundancy <b>C.</b> a deficiency <b>D.</b> an abundance


<b>15.</b> A dog ...on his owner's lap may refuse to eat from a bowl on the floor.


<b>A.</b> fed <b>B.</b> to feed <b>C.</b> is fed <b>D.</b> was fed


<b>16.</b> "Who broke the window?" - "..."


<b>A.</b> I was, thanks <b>B.</b> I did. Sorry. <b>C.</b> I did. Excuse me. <b>D.</b> I am


<b>17.</b> I had to get up early, ... I'd have missed the train.


<b>A.</b> but <b>B.</b> otherwise <b>C.</b> yet <b>D.</b> if not


<b>18.</b> "Don't tell anyone my new address" - "..."


<b>A.</b> I wouldn't <b>B.</b> I don't <b>C.</b> I won't <b>D.</b> I can't


<b>19.</b> There's a rumour that the Nation Bank is going to ... the company I work for.



<b>A.</b> take over <b>B.</b> overtake <b>C.</b> take off <b>D.</b> take on


<b>20.</b> All the cereal grains ... grow on the prairies and plains of the United States.


<b>A.</b> but rice <b>B.</b> except the rice <b>C.</b> excepting rice <b>D.</b> but for rice


<b>21.</b> " What's the biggest problem in your country?" - " We have ...of energy."


<b>A.</b> the shortage <b>B.</b> shortage <b>C.</b> a shortage <b>D.</b> some shortage


<b>22.</b> Automobile ... propane gas emits fewer dangerous pollutants into the atmosphere.


<b>A.</b> can use <b>B.</b> use <b>C.</b> using <b>D.</b> that are used


<b>23.</b> She should have been here but she's ... flu.


<b>A.</b> gone through with <b>B.</b> come up against <b>C.</b> come in for <b>D.</b> gone down with


<b>24.</b> The product was withdrawn from sale because there was no longer any ... for it.


<b>A.</b> call <b>B.</b> interest <b>C.</b> order <b>D.</b> claim


<b>25.</b> Scientists are still uncertain of ... originated millions of years ago.


<b>A.</b> about the universe <b>B.</b> which the universe <b>C.</b> with the universe <b>D.</b> how the universe


<b>26.</b> He tried to explain how he felt, but he was unable to ...his true feelings.


<b>A.</b> articulate <b>B.</b> inflict <b>C.</b> inhibit <b>D.</b> evolve



<b>27.</b> I'd like to have a bath, but there doesn't seem to be a ... or anything to stop.


<b>A.</b> plug <b>B.</b> cap <b>C.</b> lid <b>D.</b> cork


<b>28.</b> This film ... several scenes which are very funny.


<b>A.</b> illustrates <b>B.</b> pictures <b>C.</b> features <b>D.</b> depicts


<b>29.</b> How exactly did you set ...training the horses to work so well together?


<b>A.</b> empty <b>B.</b> to <b>C.</b> loose <b>D.</b> up


<b>30.</b> ... will remain the foremost world language is considered inevitable by many people.


<b>A.</b> English <b>B.</b> That English <b>C.</b> It is English that <b>D.</b> Because English


<b>31.</b> Bigamy is a situation in which a man ... two women at the same time.


<b>A.</b> is married to <b>B.</b> marries to <b>C.</b> is marry to <b>D.</b> married


<b>32.</b> "Can I use your phone?" - "..."


<b>A.</b> Of course <b>B.</b> You may not <b>C.</b> No, you can't <b>D.</b> You must


<b>33.</b> When another person sneezes, you say "..."


<b>A.</b> Pardon? <b>B.</b> Great you <b>C.</b> Bless you <b>D.</b> See you


<b>34.</b> ...the public's concern about the local environment this new road scheme will have to be


abandoned.


<b>A.</b> In the event of <b>B.</b> However much <b>C.</b> In view of <b>D.</b> As regards


<b>35.</b> - Janet: "Do you feel like going to the cinema tonight?" - Susan: "..."


</div>
<span class='text_page_counter'>(39)</span><div class='page_container' data-page=39>

<b>C.</b> I don't agree, I am afraid. <b>D.</b> That would be great


<b>36.</b> Technological advances aid in teaching, ... the basic role of teachers stays the same.


<b>A.</b> with <b>B.</b> because of <b>C.</b> but <b>D.</b> despite


<b>37.</b> No one can visit her because she has a very ...disease.


<b>A.</b> exasperating <b>B.</b> contagious <b>C.</b> intoxicating <b>D.</b> corporal


<b>38.</b> As the President was absent, I was ask to ... the meeting.


<b>A.</b> regulate <b>B.</b> govern <b>C.</b> chair <b>D.</b> officiate


<b>39.</b> By the year 2012, many people currently employed ... their jobs.


<b>A.</b> will be losing <b>B.</b> will have lost <b>C.</b> are losing <b>D.</b> have lost


<b>40.</b> I'd sooner ...deliver the new furniture tomorrow.


<b>A.</b> didn't <b>B.</b> shouldn't <b>C.</b> wouldn't <b>D.</b> mustn't


<i><b>III. Mark the letter A, B, C, or D on your answer sheet to show the underlined part that needs </b></i>
<i><b>correction. </b></i>



<b>41.</b> Organisms and their cells live by maintaining a constant exchange of elemental, ions, minerals and
gases.


<b>A.</b> constant <b>B.</b> gases <b>C.</b> by maintaining <b>D.</b> of elemental


<b>42.</b> The nests of most bird species are strategic placed to camouflage them against predators.


<b>A.</b> species <b>B.</b> against predators. <b>C.</b> most <b>D.</b> strategic


<b>43.</b> For the past few years, researchers have perfecting their control over the movements of cells and
microbes by using low power laser beams.


<b>A.</b> using <b>B.</b> their <b>C.</b> few years <b>D.</b> have perfecting


<b>44.</b> Perhaps the most unique thing about carbon atoms are their ability to combine with themselves.


<b>A.</b> the most <b>B.</b> are <b>C.</b> to combine <b>D.</b> about


<b>45.</b> Vitamin K providing the necessary impetus for the synthesis of at least two proteins involved in
blood clotting.


<b>A.</b> necessary <b>B.</b> providing <b>C.</b> involved <b>D.</b> at least


<i><b>IV. Mark the letter A, B, C, or D on your answer sheet to indicate the word that differs from the rest in </b></i>
<i><b>the position of the main stress in each of the following sentences.</b></i>


<b>46.A.</b> competitive <b>B.</b> pedestrian <b>C.</b> experienced <b>D.</b> catalogue


<b>47.A.</b> cassette <b>B.</b> police <b>C.</b> injured <b>D.</b> hotel



<b>48.A.</b> screwdriver <b>B.</b> imprudent <b>C.</b> preferably <b>D.</b> fortunately


<b>49.A.</b> government <b>B.</b> insurance <b>C.</b> envelope <b>D.</b> lavatory


<b>50.A.</b> nuclear <b>B.</b> machine <b>C.</b> refer <b>D.</b> mistake


<i><b>V. Read the following passage and mark the letter A, B, C or D on your answer sheet to indicate the </b></i>
<i><b>correct answer to each of the following questions.</b></i>


Lead poisoning in children is a major health concern. Both low and high doses of lead can have serious
effects. Children <b>exposed to</b> high doses of lead often <b>suffer</b> permanent nerve damage, mental retardation,
short attention spans, distractibility, poor academic performance, and behavioural problems. This is not a
new concern. As early as 1904, lead poisoning in children was linked to lead-based paint. Microscopic lead
particles from paint are absorbed into bloodstream when the children ingest flakes of <b>chipped</b> paint, plaster
or paint dust from suckling, or chewing on toys and other objects painted with lead-based paint. Despite
amount of lead used in their paint, this source of leading poisoning is still the most common and most
dangerous. Children living in older, <b>dilapidated</b> houses are particularly <b>at risk</b>.


<b>51.</b> What is the main topic of the passage?


<b>A.</b> Major health concern for children. <b>B.</b> Problems with household paint.


<b>C.</b> Lead poisoning in children. <b>D.</b> Lead paint in older houses.


<b>52.</b> Which of the following can be inferred from the passage?


<b>A.</b> Paint companies can no longer use lead in their paint.


</div>
<span class='text_page_counter'>(40)</span><div class='page_container' data-page=40>

<b>C.</b> Paint companies must limit the amount lead used in their paint.



<b>D.</b> Paint companies aren't required to limit the amount lead used in their paint.


<b>53.</b> According to the passage, what is the most common source of lead poisoning in children?


<b>A.</b> painted toys <b>B.</b> dilapidated houses <b>C.</b> lead-based paint <b>D.</b> household dust


<b>54.</b> Which of the following is NOT true?


<b>A.</b> Lead poisoning in children was linked to lead-based paint in the 1900s.


<b>B.</b> American paint companies today must comply with strict regulations regarding the amount of lead
used in their paint.


<b>C.</b> Only high dose of lead can have serious effects.


<b>D.</b> Lead poisoning can lead to mental retardation.


<b>55.</b> The word " <b>dilapidated " </b>is closest in meaning to which of the following?


<b>A.</b> fell down <b>B.</b> broken down <b>C.</b> poorly painted <b>D.</b> unpainted


<b>56.</b> The phrase "<b>exposed to</b>" could be best replaced by which of the following?


<b>A.</b> conducting <b>B.</b> in contact with <b>C.</b> familiar with <b>D.</b> displaying


<b>57.</b> Which of the following is closest in meaning to "<b>chipped</b>"?


<b>A.</b> sprayed <b>B.</b> fragmented <b>C.</b> unhealthy <b>D.</b> canned



<b>58.</b> What does the author imply in the final sentence of the passage?


<b>A.</b> Poor people did not comply with the regulations.


<b>B.</b> Lead-based paint chips off more easily than newer paints.


<b>C.</b> Old homes need to be rebuilt in order to be safe for children.


<b>D.</b> Old homes were painted with lead-based paint.


<b>59.</b> Which of the following is closest in meaning to the phrase "<b>at risk</b>" ?


<b>A.</b> in danger <b>B.</b> in safe way <b>C.</b> in good condition <b>D.</b> no harm


<b>60.</b> Which of the following is closest in meaning to the word "<b>suffer</b>" ?.


<b>A.</b> grieve with <b>B.</b> experience <b>C.</b> feel paint from <b>D.</b> reveal


<i><b>VI. Read the following passage and mark the letter A, B, C or D on your answer sheet to indicate the </b></i>
<i><b>correct answer to each of the following questions.</b></i>


Watching for wildlife in the forests, we rarely see past the surface of things. Standing on the ground
floor, we scan the leafy rafters, entirely overlooking the living world in the soil beneath our feet. The
forest's basement is a secret world. As different from our own world as water is from air, the soil seems
quiet, even dead. But life <b>bustles</b> down below: a cubic inch of top soil may contain billions of creatures.
Predators and prey roam beneath as well as above the forest floor. Furthermore, those upstairs and
downstairs forest denizens live closed <b>linked</b> lives. Soil-dwelling bacteria and fungi break down dead
organic matter into molecules that above ground plants use for food. Those plants as well as animals,
mature and die, leaving more organic matter to fuel the folks downstairs.



Like a well-insulated house, the soil protects its tenants from extreme temperatures, and from rain and
snow. It also provides a <b>bulwark</b> against predators that roam the surface world. But the dense, protecting
soil also limits mobility. Soil creatures must be specially equipped in order to travel easily through their
dark, <b>constricting</b> realm. Earthworms and ants are the champion earth-movers, creating channels that
allow air and water to enter the soil. While ants travel relatively far from their nests, earthworms work
small areas, <b>reprocessing </b>vast amounts of soil into fertile "waste". In a single year, as much as 36 tons of
soil may pass through the alimentary tracts of all the earthworms living in an acre of soil.


<b>61.</b> It can be inferred from the passage that the forest soil is ...


<b>A.</b> unknown to man <b>B.</b> densely inhabited <b>C.</b> sparely inhabited <b>D.</b> devoid of life


<b>62.</b> The word "<b>bustles</b>" in the passage is closest in meaning to ...


<b>A.</b> continues <b>B.</b> waits <b>C.</b> threatens <b>D.</b> is very active


<b>63.</b> The main topic of the passage is ...


<b>A.</b> a description of a forest scene <b>B.</b> the life cycle of ants and worms


<b>C.</b> the habits of the forest animals. <b>D.</b> life in the forest soil


<b>64.</b> The soil offers creatures that live underground protection EXPECT ...


</div>
<span class='text_page_counter'>(41)</span><div class='page_container' data-page=41>

<b>65.</b> The word "<b>reprocessing</b>" in the passage is closest in meaning to ...


<b>A.</b> transporting <b>B.</b> eliminating <b>C.</b> arranging <b>D.</b> converting


<b>66.</b> The word " <b>bulwark</b>" in the passage is closest in meaning to ...



<b>A.</b> tomb <b>B.</b> gateway <b>C.</b> barrier <b>D.</b> radar


<b>67.</b> The word "<b>constricting</b>" in the passage is closest in meaning to ...


<b>A.</b> damp <b>B.</b> deep <b>C.</b> heavy <b>D.</b> limiting


<b>68.</b> It could be expected that ants ...


<b>A.</b> perform similar functions to fungi <b>B.</b> move more earth than earthworms


<b>C.</b> live only above ground <b>D.</b> are more mobile than earthworms


<b>69.</b> The word "<b>linked</b>" in the passage is closest in meaning to ...


<b>A.</b> related <b>B.</b> mechanical <b>C.</b> measured <b>D.</b> chained


<b>70.</b> According to the passage, what is the main function of bacteria and fungi?


<b>A.</b> to kill mature plants <b>B.</b> to provide food for plant life


<b>C.</b> to help aerate the soil <b>D.</b> to build walls in the soil


<i><b>VII. Read the following passage and mark the letter A, B, C or D on your answer sheet to indicate the </b></i>
<i><b>correct word for each of the blanks.</b></i>


In this age of ...(71)... telephone networks and electronic mail, it seems that fewer and even fewer
people are taking time to sit down and write letters ...(72)... friends and relatives. For hundreds of years,
letters were the only way to keep ...(73)... people who were any distance away and letter-writing was seen
as an important skill for all learned people...(74).... Gradually, ...(75)..., the importance of writing letters is
decreasing to a point that majority of us have to ...(76)... a special effort to turn out something worthwhile


when we apply for a job or make an appointment. In business circles the tendency is for routine
communications to become shorter. ...(77)...clients may appreciate a detailed letter, an employee who
sends out long letter is often regarded as ...(78).... Many people prefer the telephone in all circumstances
and its speed is essential in many situations but ...(79)... have you put the telephone down, dissatisfied
with what you have managed to say? I don't think I'll throw my ...(80)... away yet.


<b>71.A.</b> all are correct <b>B.</b> highly-developed <b>C.</b> advanced <b>D.</b> progressive


<b>72.A.</b> to <b>B.</b> with <b>C.</b> from <b>D.</b> for


<b>73.A.</b> in step with <b>B.</b> in favour of <b>C.</b> on good terms with <b>D.</b> in contact with


<b>74.A.</b> mastering <b>B.</b> mastered <b>C.</b> to master <b>D.</b> to be mastered


<b>75.A.</b> for example <b>B.</b> therefore <b>C.</b> however <b>D.</b> in short


<b>76.A.</b> cause <b>B.</b> create <b>C.</b> make <b>D.</b> do


<b>77.A.</b> However <b>B.</b> As though <b>C.</b> Even though <b>D.</b> Despite


<b>78.A.</b> impossible <b>B.</b> unimportant <b>C.</b> inefficient <b>D.</b> unusual


<b>79.A.</b> how much <b>B.</b> how often <b>C.</b> how about <b>D.</b> how long


</div>
<span class='text_page_counter'>(42)</span><div class='page_container' data-page=42>

<i>Đáp án đề: 127</i>


<b> 01. </b>- | - - <b> 21. </b>- - } - <b> 41. </b>- - - ~ <b> 61. </b>- | - -


<b> 02. </b>- - - ~ <b> 22. </b>- - } - <b> 42. </b>- - - ~ <b> 62. </b>- - - ~



<b> 03. </b>{ - - - <b> 23. </b>- - - ~ <b> 43. </b>- - - ~ <b> 63. </b>- - - ~


<b> 04. </b>{ - - - <b> 24. </b>{ - - - <b> 44. </b>- | - - <b> 64. </b>- | - -


<b> 05. </b>- - } - <b> 25. </b>- - - ~ <b> 45. </b>- | - - <b> 65. </b>- - - ~


<b> 06. </b>- | - - <b> 26. </b>{ - - - <b> 46. </b>- - - ~ <b> 66. </b>- - } -


<b> 07. </b>- - - ~ <b> 27. </b>{ - - - <b> 47. </b>- - - ~ <b> 67. </b>- - - ~


<b> 08. </b>- - } - <b> 28. </b>- - } - <b> 48. </b>- | - - <b> 68. </b>- - - ~


<b> 09. </b>- - - ~ <b> 29. </b>- - - ~ <b> 49. </b>- | - - <b> 69. </b>{ - - -


<b> 10. </b>{ - - - <b> 30. </b>- | - - <b> 50. </b>{ - - - <b> 70. </b>- | - -


<b> 11. </b>- - - ~ <b> 31. </b>{ - - - <b> 51. </b>- - } - <b> 71. </b>{ - - -


<b> 12. </b>- | - - <b> 32. </b>{ - - - <b> 52. </b>- - } - <b> 72. </b>{ - - -


<b> 13. </b>- - - ~ <b> 33. </b>- - } - <b> 53. </b>- - } - <b> 73. </b>- - - ~


<b> 14. </b>- - - ~ <b> 34. </b>- - } - <b> 54. </b>- - } - <b> 74. </b>- - } -


<b> 15. </b>{ - - - <b> 35. </b>- - - ~ <b> 55. </b>- | - - <b> 75. </b>- - } -


<b> 16. </b>- | - - <b> 36. </b>- - } - <b> 56. </b>- | - - <b> 76. </b>- - } -


<b> 17. </b>- | - - <b> 37. </b>- | - - <b> 57. </b>- | - - <b> 77. </b>- - } -



<b> 18. </b>- - } - <b> 38. </b>- - } - <b> 58. </b>- - } - <b> 78. </b>- - } -


<b> 19. </b>{ - - - <b> 39. </b>- | - - <b> 59. </b>{ - - - <b> 79. </b>- | - -


</div>
<span class='text_page_counter'>(43)</span><div class='page_container' data-page=43>

TRƯỜNG THPT LÊ HỒNG PHONG <b> KỲ THI THỬ ĐẠI HỌC - LẦN II</b>
<b> Thị xã Bỉm Sơn </b> <b> NĂM HỌC 2010-2011</b>


<b> </b> <b> </b> <b> Môn thi: Tiếng Anh</b>


<b> Thời gian làm bài: 90 phút</b>
<b> ĐỀ CHÍNH THỨC</b>


<b> (đề thi gồm 5 trang)</b> <b>Mã đề: 128</b>
<b> </b>


Họ và tên thí sinh:... Số báo danh:...


<i><b>I. Mark the letter A, B, C, or D on your answer sheet to indicate the word that differs from the rest in </b></i>
<i><b>the position of the main stress in each of the following sentences.</b></i>


<b>01.A.</b> lavatory <b>B.</b> envelope <b>C.</b> insurance <b>D.</b> government


<b>02.A.</b> catalogue <b>B.</b> experienced <b>C.</b> competitive <b>D.</b> pedestrian


<b>03.A.</b> refer <b>B.</b> mistake <b>C.</b> machine <b>D.</b> nuclear


<b>04.A.</b> injured <b>B.</b> hotel <b>C.</b> police <b>D.</b> cassette


<b>05.A.</b> fortunately <b>B.</b> imprudent <b>C.</b> preferably <b>D.</b> screwdriver



<i><b>II. Read the following passage and mark the letter A, B, C or D on your answer sheet to indicate the </b></i>
<i><b>correct word for each of the blanks.</b></i>


In this age of ...(6)... telephone networks and electronic mail, it seems that fewer and even fewer
people are taking time to sit down and write letters ...(7)... friends and relatives. For hundreds of years,
letters were the only way to keep ...(8)... people who were any distance away and letter-writing was seen as
an important skill for all learned people...(9).... Gradually, ...(10)..., the importance of writing letters is
decreasing to a point that majority of us have to ...(11)... a special effort to turn out something worthwhile
when we apply for a job or make an appointment. In business circles the tendency is for routine
communications to become shorter. ...(12)...clients may appreciate a detailed letter, an employee who
sends out long letter is often regarded as ...(13).... Many people prefer the telephone in all circumstances
and its speed is essential in many situations but ...(14)... have you put the telephone down, dissatisfied
with what you have managed to say? I don't think I'll throw my ...(15)... away yet.


<b>06.A.</b> highly-developed <b>B.</b> advanced <b>C.</b> all are correct <b>D.</b> progressive


<b>07.A.</b> with <b>B.</b> from <b>C.</b> to <b>D.</b> for


<b>08.A.</b> on good terms with <b>B.</b> in favour of <b>C.</b> in step with <b>D.</b> in contact with


<b>09.A.</b> to master <b>B.</b> mastered <b>C.</b> mastering <b>D.</b> to be mastered


<b>10.A.</b> in short <b>B.</b> however <b>C.</b> for example <b>D.</b> therefore


<b>11.A.</b> cause <b>B.</b> create <b>C.</b> make <b>D.</b> do


<b>12.A.</b> Despite <b>B.</b> As though <b>C.</b> Even though <b>D.</b> However


<b>13.A.</b> inefficient <b>B.</b> unimportant <b>C.</b> unusual <b>D.</b> impossible



<b>14.A.</b> how long <b>B.</b> how about <b>C.</b> how much <b>D.</b> how often


<b>15.A.</b> effort <b>B.</b> letter <b>C.</b> pen <b>D.</b> telephone


<i><b>III. Mark the letter A, B, C, or D on your answer sheet to show the underlined part that needs </b></i>
<i><b>correction. </b></i>


<b>16.</b> Perhaps the most unique thing about carbon atoms are their ability to combine with themselves.


<b>A.</b> are <b>B.</b> to combine <b>C.</b> the most <b>D.</b> about


<b>17.</b> For the past few years, researchers have perfecting their control over the movements of cells and
microbes by using low power laser beams.


<b>A.</b> have perfecting <b>B.</b> few years <b>C.</b> using <b>D.</b> their


<b>18.</b> Vitamin K providing the necessary impetus for the synthesis of at least two proteins involved in
blood clotting.


<b>A.</b> necessary <b>B.</b> providing <b>C.</b> at least <b>D.</b> involved


<b>19.</b> The nests of most bird species are strategic placed to camouflage them against predators.


</div>
<span class='text_page_counter'>(44)</span><div class='page_container' data-page=44>

<b>20.</b> Organisms and their cells live by maintaining a constant exchange of elemental, ions, minerals and
gases.


<b>A.</b> by maintaining <b>B.</b> gases <b>C.</b> constant <b>D.</b> of elemental


<i><b>IV. Read the following passage and mark the letter A, B, C or D on your answer sheet to indicate the </b></i>
<i><b>correct answer to each of the following questions.</b></i>



Lead poisoning in children is a major health concern. Both low and high doses of lead can have serious
effects. Children <b>exposed to</b> high doses of lead often <b>suffer</b> permanent nerve damage, mental retardation,
short attention spans, distractibility, poor academic performance, and behavioural problems. This is not a
new concern. As early as 1904, lead poisoning in children was linked to lead-based paint. Microscopic lead
particles from paint are absorbed into bloodstream when the children ingest flakes of <b>chipped</b> paint, plaster
or paint dust from suckling, or chewing on toys and other objects painted with lead-based paint. Despite
amount of lead used in their paint, this source of leading poisoning is still the most common and most
dangerous. Children living in older, <b>dilapidated</b> houses are particularly <b>at risk</b>.


<b>21.</b> The phrase "<b>exposed to</b>" could be best replaced by which of the following?


<b>A.</b> conducting <b>B.</b> familiar with <b>C.</b> in contact with <b>D.</b> displaying


<b>22.</b> What does the author imply in the final sentence of the passage?


<b>A.</b> Old homes were painted with lead-based paint.


<b>B.</b> Old homes need to be rebuilt in order to be safe for children.


<b>C.</b> Poor people did not comply with the regulations.


<b>D.</b> Lead-based paint chips off more easily than newer paints.


<b>23.</b> Which of the following is NOT true?


<b>A.</b> Only high dose of lead can have serious effects.


<b>B.</b> Lead poisoning can lead to mental retardation.



<b>C.</b> Lead poisoning in children was linked to lead-based paint in the 1900s.


<b>D.</b> American paint companies today must comply with strict regulations regarding the amount of lead
used in their paint.


<b>24.</b> What is the main topic of the passage?


<b>A.</b> Major health concern for children. <b>B.</b> Lead poisoning in children.


<b>C.</b> Lead paint in older houses. <b>D.</b> Problems with household paint.


<b>25.</b> Which of the following is closest in meaning to "<b>chipped</b>"?


<b>A.</b> unhealthy <b>B.</b> sprayed <b>C.</b> canned <b>D.</b> fragmented


<b>26.</b> The word " <b>dilapidated " </b>is closest in meaning to which of the following?


<b>A.</b> unpainted <b>B.</b> broken down <b>C.</b> fell down <b>D.</b> poorly painted


<b>27.</b> Which of the following is closest in meaning to the phrase "<b>at risk</b>" ?


<b>A.</b> in good condition <b>B.</b> no harm <b>C.</b> in safe way <b>D.</b> in danger


<b>28.</b> According to the passage, what is the most common source of lead poisoning in children?


<b>A.</b> lead-based paint <b>B.</b> painted toys <b>C.</b> dilapidated houses <b>D.</b> household dust


<b>29.</b> Which of the following can be inferred from the passage?


<b>A.</b> Paint companies must limit the amount lead used in their paint.



<b>B.</b> Paint companies aren't required to limit the amount lead used in their paint.


<b>C.</b> Paint companies can no longer use lead in their paint.


<b>D.</b> Paint companies have always followed regulations regarding the amount of lead used in their paint.


<b>30.</b> Which of the following is closest in meaning to the word "<b>suffer</b>" ?.


<b>A.</b> feel paint from <b>B.</b> experience <b>C.</b> reveal <b>D.</b> grieve with


<b>V.Mark the letter A, B, C or D on your answer sheet to indicate the sentence that is closest in </b>
<b>meaning to each of following sentences.</b>


<b>31.</b> She started work three months ago.


<b>A.</b> It's three months that she worked for. <b>B.</b> She had been working for three months.


<b>C.</b> She is working here for three months now. <b>D.</b> It is three months since she started work.


<b>32.</b> This is the most delicious meat I've ever eaten.


</div>
<span class='text_page_counter'>(45)</span><div class='page_container' data-page=45>

<b>B.</b> Not ever in my life have I had such a wonderful meat.


<b>C.</b> Never in my life have I had such a delicious meat


<b>D.</b> I have never eaten such more delicious meat.


<b>33.</b> "How beautiful is the dress you have just bought!" Peter said to Mary.



<b>A.</b> Peter complimented Mary on her beautiful dress.


<b>B.</b> Peter said thanks to Mary for her beautiful dress


<b>C.</b> Peter promised to buy Mary a beautiful dress.


<b>D.</b> Peter asked Mary how she had just bought her beautiful dress.


<b>34.</b> We came home early to avoid the coming storm.


<b>A.</b> Because it was predicted storming, we decided to come home earlier than unusual.


<b>B.</b> We didn't come home late to not get the coming storm.


<b>C.</b> In order to avoiding the coming storm, we came home early.


<b>D.</b> We came home early in order to avoid the coming storm.


<b>35.</b> My career as a teacher began 14 years ago.


<b>A.</b> I was a teacher for 14 years. <b>B.</b> I have been a teacher for 14 years now.


<b>C.</b> I have started teaching for 14 years now. <b>D.</b> For 14 years have I been a teacher.


<b>36.</b> My father regretted working for that company.


<b>A.</b> My father wished he hadn't worked for that company.


<b>B.</b> It is my father's regret working for that company.



<b>C.</b> My father didn't like working for that company.


<b>D.</b> If only my father was not working for that company.


<b>37.</b> This affair does not concern you.


<b>A.</b> Don't do this affair. <b>B.</b> Your concern is to do this affair.


<b>C.</b> This affair is no business of yours. <b>D.</b> This affair is not interesting.


<b>38.</b> Not many people attended the meeting.


<b>A.</b> Too many people turned out at the meeting. <b>B.</b> No people came to the meeting


<b>C.</b> People didn't show up at the meeting. <b>D.</b> There was a poor turn-out for the meeting.


<b>39.</b> My father finds maps hard to follow.


<b>A.</b> Map-reading is not interesting to my father at all.


<b>B.</b> My father always gets lost.


<b>C.</b> My father has trouble following maps.


<b>D.</b> My father can't read maps at all.


<b>40.</b> She has lost her appetite recently.


<b>A.</b> She has gone off food recently. <b>B.</b> Her appetite has been very good.



<b>C.</b> She has eaten a lot of food recently. <b>D.</b> She hasn't had any food recently.


<i><b>VI. Mark the letter A, B, C, or D on your answer sheet to indicate the correct answer to each of the </b></i>
<i><b>following sentences</b></i>


<b>41.</b> I'd sooner ...deliver the new furniture tomorrow.


<b>A.</b> didn't <b>B.</b> mustn't <b>C.</b> wouldn't <b>D.</b> shouldn't


<b>42.</b> Scientists are still uncertain of ... originated millions of years ago.


<b>A.</b> with the universe <b>B.</b> about the universe <b>C.</b> how the universe <b>D.</b> which the universe


<b>43.</b> She should have been here but she's ... flu.


<b>A.</b> come in for <b>B.</b> gone through with <b>C.</b> gone down with <b>D.</b> come up against


<b>44.</b> How exactly did you set ...training the horses to work so well together?


<b>A.</b> up <b>B.</b> to <b>C.</b> empty <b>D.</b> loose


<b>45.</b> As the President was absent, I was ask to ... the meeting.


<b>A.</b> officiate <b>B.</b> chair <b>C.</b> regulate <b>D.</b> govern


<b>46.</b> All the cereal grains ... grow on the prairies and plains of the United States.


<b>A.</b> excepting rice <b>B.</b> but for rice <b>C.</b> except the rice <b>D.</b> but rice


<b>47.</b> I'd like to have a bath, but there doesn't seem to be a ... or anything to stop.



</div>
<span class='text_page_counter'>(46)</span><div class='page_container' data-page=46>

<b>48.</b> "Who broke the window?" - "..."


<b>A.</b> I am <b>B.</b> I did. Sorry. <b>C.</b> I was, thanks <b>D.</b> I did. Excuse me.


<b>49.</b> " What's the biggest problem in your country?" - " We have ...of energy."


<b>A.</b> shortage <b>B.</b> a shortage <b>C.</b> the shortage <b>D.</b> some shortage


<b>50.</b> This film ... several scenes which are very funny.


<b>A.</b> illustrates <b>B.</b> features <b>C.</b> depicts <b>D.</b> pictures


<b>51.</b> Only when a great deal of more information has been obtained, ...to plan a manned trip to the
planet.


<b>A.</b> will it possible <b>B.</b> it will be possible <b>C.</b> it will be likely <b>D.</b> it will probably be


<b>52.</b> There's a rumour that the Nation Bank is going to ... the company I work for.


<b>A.</b> overtake <b>B.</b> take off <b>C.</b> take over <b>D.</b> take on


<b>53.</b> Technological advances aid in teaching, ... the basic role of teachers stays the same.


<b>A.</b> with <b>B.</b> because of <b>C.</b> despite <b>D.</b> but


<b>54.</b> ... will remain the foremost world language is considered inevitable by many people.


<b>A.</b> That English <b>B.</b> English <b>C.</b> Because English <b>D.</b> It is English that



<b>55.</b> "Can I use your phone?" - "..."


<b>A.</b> Of course <b>B.</b> You must <b>C.</b> No, you can't <b>D.</b> You may not


<b>56.</b> Bigamy is a situation in which a man ... two women at the same time.


<b>A.</b> is marry to <b>B.</b> married <b>C.</b> is married to <b>D.</b> marries to


<b>57.</b> A child in the first grade tends to be ... all of the other children in the class.


<b>A.</b> the same old to <b>B.</b> the same age than <b>C.</b> the same age as <b>D.</b> as old like


<b>58.</b> Automobile ... propane gas emits fewer dangerous pollutants into the atmosphere.


<b>A.</b> using <b>B.</b> use <b>C.</b> can use <b>D.</b> that are used


<b>59.</b> The product was withdrawn from sale because there was no longer any ... for it.


<b>A.</b> call <b>B.</b> interest <b>C.</b> order <b>D.</b> claim


<b>60.</b> "Don't tell anyone my new address" - "..."


<b>A.</b> I don't <b>B.</b> I wouldn't <b>C.</b> I can't <b>D.</b> I won't


<b>61.</b> Their climate is not dry at all; in fact, they have ...of water.


<b>A.</b> a redundancy <b>B.</b> an abundance <b>C.</b> a deficiency <b>D.</b> a conglomeration


<b>62.</b> I had to get up early, ... I'd have missed the train.



<b>A.</b> otherwise <b>B.</b> but <b>C.</b> if not <b>D.</b> yet


<b>63.</b> When another person sneezes, you say "..."


<b>A.</b> See you <b>B.</b> Pardon? <b>C.</b> Great you <b>D.</b> Bless you


<b>64.</b> He tried to explain how he felt, but he was unable to ...his true feelings.


<b>A.</b> evolve <b>B.</b> inhibit <b>C.</b> articulate <b>D.</b> inflict


<b>65.</b> No one can visit her because she has a very ...disease.


<b>A.</b> corporal <b>B.</b> intoxicating <b>C.</b> exasperating <b>D.</b> contagious


<b>66.</b> A dog ...on his owner's lap may refuse to eat from a bowl on the floor.


<b>A.</b> fed <b>B.</b> to feed <b>C.</b> was fed <b>D.</b> is fed


<b>67.</b> ...the public's concern about the local environment this new road scheme will have to be
abandoned.


<b>A.</b> However much <b>B.</b> In view of <b>C.</b> In the event of <b>D.</b> As regards


<b>68.</b> - Janet: "Do you feel like going to the cinema tonight?" - Susan: "..."


<b>A.</b> I feel very bored. <b>B.</b> I don't agree, I am afraid.


<b>C.</b> That would be great <b>D.</b> You're welcome


<b>69.</b> By the year 2012, many people currently employed ... their jobs.



<b>A.</b> will be losing <b>B.</b> are losing <b>C.</b> will have lost <b>D.</b> have lost


<b>70.</b> That popular magazine has many ...


</div>
<span class='text_page_counter'>(47)</span><div class='page_container' data-page=47>

<i><b>VII. Read the following passage and mark the letter A, B, C or D on your answer sheet to indicate the </b></i>
<i><b>correct answer to each of the following questions.</b></i>


Watching for wildlife in the forests, we rarely see past the surface of things. Standing on the ground
floor, we scan the leafy rafters, entirely overlooking the living world in the soil beneath our feet. The
forest's basement is a secret world. As different from our own world as water is from air, the soil seems
quiet, even dead. But life <b>bustles</b> down below: a cubic inch of top soil may contain billions of creatures.
Predators and prey roam beneath as well as above the forest floor. Furthermore, those upstairs and
downstairs forest denizens live closed <b>linked</b> lives. Soil-dwelling bacteria and fungi break down dead
organic matter into molecules that above ground plants use for food. Those plants as well as animals,
mature and die, leaving more organic matter to fuel the folks downstairs.


Like a well-insulated house, the soil protects its tenants from extreme temperatures, and from rain and
snow. It also provides a <b>bulwark</b> against predators that roam the surface world. But the dense, protecting
soil also limits mobility. Soil creatures must be specially equipped in order to travel easily through their
dark, <b>constricting</b> realm. Earthworms and ants are the champion earth-movers, creating channels that
allow air and water to enter the soil. While ants travel relatively far from their nests, earthworms work
small areas, <b>reprocessing </b>vast amounts of soil into fertile "waste". In a single year, as much as 36 tons of
soil may pass through the alimentary tracts of all the earthworms living in an acre of soil.


<b>71.</b> The main topic of the passage is ...


<b>A.</b> life in the forest soil <b>B.</b> a description of a forest scene


<b>C.</b> the habits of the forest animals. <b>D.</b> the life cycle of ants and worms



<b>72.</b> It could be expected that ants ...


<b>A.</b> move more earth than earthworms <b>B.</b> are more mobile than earthworms


<b>C.</b> live only above ground <b>D.</b> perform similar functions to fungi


<b>73.</b> The word "<b>linked</b>" in the passage is closest in meaning to ...


<b>A.</b> mechanical <b>B.</b> related <b>C.</b> chained <b>D.</b> measured


<b>74.</b> It can be inferred from the passage that the forest soil is ...


<b>A.</b> devoid of life <b>B.</b> unknown to man <b>C.</b> sparely inhabited <b>D.</b> densely inhabited


<b>75.</b> The soil offers creatures that live underground protection EXPECT ...


<b>A.</b> bad weather <b>B.</b> bacteria and fungi <b>C.</b> enemies <b>D.</b> extreme heat and cold


<b>76.</b> According to the passage, what is the main function of bacteria and fungi?


<b>A.</b> to help aerate the soil <b>B.</b> to kill mature plants


<b>C.</b> to build walls in the soil <b>D.</b> to provide food for plant life


<b>77.</b> The word " <b>bulwark</b>" in the passage is closest in meaning to ...


<b>A.</b> barrier <b>B.</b> radar <b>C.</b> gateway <b>D.</b> tomb


<b>78.</b> The word "<b>reprocessing</b>" in the passage is closest in meaning to ...



<b>A.</b> arranging <b>B.</b> converting <b>C.</b> eliminating <b>D.</b> transporting


<b>79.</b> The word "<b>constricting</b>" in the passage is closest in meaning to ...


<b>A.</b> deep <b>B.</b> damp <b>C.</b> limiting <b>D.</b> heavy


<b>80.</b> The word "<b>bustles</b>" in the passage is closest in meaning to ...


</div>
<span class='text_page_counter'>(48)</span><div class='page_container' data-page=48>

<i>Đáp án đề: 128</i>


<b> 01. </b>- - } - <b> 21. </b>- - } - <b> 41. </b>- - - ~ <b> 61. </b>- | - -


<b> 02. </b>{ - - - <b> 22. </b>- | - - <b> 42. </b>- - } - <b> 62. </b>{ - - -


<b> 03. </b>- - - ~ <b> 23. </b>{ - - - <b> 43. </b>- - } - <b> 63. </b>- - - ~


<b> 04. </b>- | - - <b> 24. </b>- | - - <b> 44. </b>{ - - - <b> 64. </b>- - } -


<b> 05. </b>- | - - <b> 25. </b>- - - ~ <b> 45. </b>- | - - <b> 65. </b>- - - ~


<b> 06. </b>- - } - <b> 26. </b>- | - - <b> 46. </b>- - - ~ <b> 66. </b>{ - - -


<b> 07. </b>- - } - <b> 27. </b>- - - ~ <b> 47. </b>{ - - - <b> 67. </b>- | - -


<b> 08. </b>- - - ~ <b> 28. </b>{ - - - <b> 48. </b>- | - - <b> 68. </b>- - } -


<b> 09. </b>{ - - - <b> 29. </b>{ - - - <b> 49. </b>- | - - <b> 69. </b>- - } -


<b> 10. </b>- | - - <b> 30. </b>- | - - <b> 50. </b>- | - - <b> 70. </b>- - } -



<b> 11. </b>- - } - <b> 31. </b>- - - ~ <b> 51. </b>{ - - - <b> 71. </b>{ - - -


<b> 12. </b>- - } - <b> 32. </b>- - - ~ <b> 52. </b>- - } - <b> 72. </b>- | - -


<b> 13. </b>{ - - - <b> 33. </b>{ - - - <b> 53. </b>- - - ~ <b> 73. </b>- | - -


<b> 14. </b>- - - ~ <b> 34. </b>- - - ~ <b> 54. </b>{ - - - <b> 74. </b>- - - ~


<b> 15. </b>- - } - <b> 35. </b>- | - - <b> 55. </b>{ - - - <b> 75. </b>- | - -


<b> 16. </b>{ - - - <b> 36. </b>{ - - - <b> 56. </b>- - } - <b> 76. </b>- - - ~


<b> 17. </b>{ - - - <b> 37. </b>- - } - <b> 57. </b>- - } - <b> 77. </b>{ - - -


<b> 18. </b>- | - - <b> 38. </b>- - - ~ <b> 58. </b>{ - - - <b> 78. </b>- | - -


<b> 19. </b>- - - ~ <b> 39. </b>- - } - <b> 59. </b>{ - - - <b> 79. </b>- - } -


</div>

<!--links-->

Tài liệu bạn tìm kiếm đã sẵn sàng tải về

Tải bản đầy đủ ngay
×